PDA

توجه ! این یک نسخه آرشیو شده میباشد و در این حالت شما عکسی را مشاهده نمیکنید برای مشاهده کامل متن و عکسها بر روی لینک مقابل کلیک کنید : رفع اشكال المپیاد نجوم



صفحه ها : 1 2 [3]

albertini
10-24-2013, 12:36 PM
فقط کافیه st هر دو جسم در زمان طلوع یکسان باشه.
سلام جناب انصارين

ميشه درباره ي پارامتر st كمي توضيح بديد

ممنون

univers
10-24-2013, 12:48 PM
St یا همان زمان نجومی برابر است با زاویه ساعتی ستاره (h)به علاوه ی بعد آن ستاره(ra یا آلفا) که میتوان نشان داد که برابر است با زاویه ساعتی اعتدال بهاری .

ali zeynali
10-24-2013, 09:35 PM
St یا همان زمان نجومی برابر است با زاویه ساعتی ستاره (h)به علاوه ی بعد آن ستاره(ra یا آلفا) که میتوان نشان داد که برابر است با زاویه ساعتی اعتدال بهاری .
برعکس یعنی زمان نجومی رو زاویه ساعتی نقطه اول حمل تعریف می کنند بعد نشون می دن که با مجموع زاویه ساعتی و بعد هر ستاره ای برابر ه.

behnood
11-04-2013, 10:25 PM
سلام دوستان
یه سوال خیلی ابتدایی داشتم ...

یک خوشهي ستاره اي شامل بینهایت ستاره داریم. 1 ستاره قدر 1، 2 ستاره قدر 2، 3 ستاره قدر 3 و الی آخر. قدر مجموع چند میشه و چطوری حساب میشه


این سوال رو پارسال حل کرده بودم الان یادم رفته اگه راهنمایی کنین ممنون میشم ...

A.ALAVI
11-04-2013, 10:56 PM
سلام دوستان
یه سوال خیلی ابتدایی داشتم ...

یک خوشهي ستاره اي شامل بینهایت ستاره داریم. 1 ستاره قدر 1، 2 ستاره قدر 2، 3 ستاره قدر 3 و الی آخر. قدر مجموع چند میشه و چطوری حساب میشه


این سوال رو پارسال حل کرده بودم الان یادم رفته اگه راهنمایی کنین ممنون میشم ...

اوّل اینکه با استفاده از تعریف هیپارخوس از قدر ها میدونیم که ستاره ی قدر 1؛100 برابر درخشنده تر از ستاره ی قدر 6! پس ستاره ی قدر یک (1/2.5)^10 برابر درخشنده تر از ستاره ی قدر 2 و ستاره ی قدر 2 (1/2.5)^10 از قدر 3 و به همین ترتیب...
پس میتونیم درخشندگی همشونو بر حسب همون ستاره ی قدر 1 بنویسیم که یه تصاعد هندسی مانندی میشه که بدست میاد درخشندگی مجموع، تقریباً 2.760 برابر درخشندگی ستاره ی قدر 1 که با نوشتن فرمول قدر بدست میاریم که قدر مطلق مجموع این خوشه برابر با: 0.102-
الان اثبات اون فرمول ریاضی رو هم مینویسم!

http://up.avastarco.com/images/lr3jcgo0oup76hl4zkd1.png (http://up.avastarco.com/)

behnood
11-04-2013, 11:52 PM
علی دمت گرم
طلا گوارای وجودت :)

ali s.k
11-05-2013, 10:23 PM
سلام
لطفا این جمله رو توضیح بدین:شدت تابش به انرژی عبوری از یک عنصر سطحی dA در زاویه فضایی dω در راستای θ مربوط می شود.
اگه ممکنه تفاوت بین شار و شدت رو هم بگین.
با سپاس.

Hooman_G
11-06-2013, 09:02 AM
سلام
لطفا این جمله رو توضیح بدین:شدت تابش به انرژی عبوری از یک عنصر سطحی dA در زاویه فضایی dω در راستای θ مربوط می شود.
اگه ممکنه تفاوت بین شار و شدت رو هم بگین.
با سپاس.

سلام،
شار : اگر قابي به مساحت ١*١ رو مقابل هر كميت اي كه در حال حركت است بذاريم و در فاصله ي زماني ١ ثانيه مقداري كه از اون كميت از قاب گذشته رو حساب كنيم، شار اون كميت رو ميده.
مثلا شار اندازه حركت( تكانه) برابر با فشاره.
شدت:
دستگاه مختصاتي رو رسم مي كنم كه صفحه ي xy ان از إلمان dA بگذره. يك المان (d(omega هم در فاصله ي(theta) از محور z وفاصله ي ( phi) از محور x رسم مي كنم.
حالا فرض كنيد كه پرتو ي نوري با طول موج (lambda) و((llambda)+d(lambda)) از اين سطح عبور كنه .ما ميخوايم شار گذرنده از اين زاويه فضايي رو بدست بياريم . اگر dA رو در cos theta ضرب كنم ، تصويرش رو روي سطح عمودي اي پيدا ميكنم كه پرتو هاي نور در اون راستا در حركتند.
حالا طبق گفته ي اوليه اگر بيام و انرژي رو بر dOmega بر dt بر dlambda بر dA cos theta تقسيم كنم، شارش رو در ديفرانسيل زاويه فضايي پيدا كردم كه اين همون شدته
در واقع به زبون ساده شدت ، شار بر واحد زاويه فضاييه . تفاوت شهوديش اينه كه شار (flux) برابر با كل روشنايي يك ستارس در حالي كه شدت روشنايي(brightnesd) رو در يك راستاي خاص هدايت مي كنه.  

mjaasgari
11-06-2013, 07:30 PM
سلام
خسته نباشید
یه سوال:توی کاتالوگ های نجومی بعد و میل ستاره ها رو در چه حالتی که ستاره توی اون حالت قرار داره مینویسند؟
یعنی اگر بعد و میل اندازه گیری شده توسط ما با بعد و میل کاتالوگ یکی باشه ویژگی ستاره چیه؟
ممنون

ali s.k
11-06-2013, 09:43 PM
سلام
چرا مساحت زیر نمودار پلانک برابر شار کل می باشد؟اچگونه می توان شار در یک طول موج خاص رو از روی نمودار پلانک بدست اورد؟
با تشکر.

Amirali
11-06-2013, 10:49 PM
سلام
خسته نباشید
یه سوال:توی کاتالوگ های نجومی بعد و میل ستاره ها رو در چه حالتی که ستاره توی اون حالت قرار داره مینویسند؟
یعنی اگر بعد و میل اندازه گیری شده توسط ما با بعد و میل کاتالوگ یکی باشه ویژگی ستاره چیه؟
ممنون

سلام
من منظورتون رو دقیقا نفهمیدم!
بعد و میل یه چیز ثابته و تغییر نمیکنه ،البته در طولانی مدت با توجه به حرکت خود ستاره و حرکت تقدیمی زمین و... تغییر میکنه
اما الان ما بعد و میل رو استاندارد سال 2000 در نظر میگیریم و تا مدت طولانی هم همین اعداد با دقت زیادی قابل قبول هستن
اگه بعد میل اندازه گیری شده توسط ما با کاتالوگ یکی باشه نتیجش اینه که با دقت خوبی بعد و میل رو اندازه گرفتیم ;)

Hooman_G
11-07-2013, 03:17 AM
سلام
چرا مساحت زیر نمودار پلانک برابر شار کل می باشد؟اچگونه می توان شار در یک طول موج خاص رو از روی نمودار پلانک بدست اورد؟
با تشکر.
بازم سلام!
ببينيد، نمودار پلانك يك نمودار توزيع و تابعش هم يك تابع توزيع هست. تابع توزيع يعني از هر كميتي(x) چند تا(f(x داريم
براي اينكه بهتر متوجه بشين يه مثال ميزنم: فرض كنيد تابع توزيع جرم يك خوشه ستاره رو داده باشن، يعني بگن از هر
جرمي توي اون خوشه چند تا هست. حالا قبول دارين كه اگر جرم هر ستاره رو ضرب در تعدادش كنيم بعد با بعدي جمع بزنيم همينجور تا اخر جرم كل خوشه رو ميده؟ حالا اگه حد اين سيگما رو به سمت بي نهايت و مقدار جرم رو به بازه هاي كوچك ( ديفرانسيلي) تقسيم كنيم ميشه انتگرال اون تابع از ٠ تا بي نهايت . توي نمودار پلانك هم عملا شار در هر طول- موجو تو طول موجش ضرب مي كنيم. كه اينم مثل قبلي ميشه انتگرال از ٠ تا بي نهايت (.B(lam.)d(lam . حالا اگه انتگرال رو تو محدوده خاص بخواين بگيرين ، مثلا ٤٥٠ تا ٥٥٠ نانو متر دو راه هست ؛ اولي اينكه انتگرال رو تو اون بازه بگيريد
دومي اينكه اگر مثل الان بازه زياد بزرگ نبود، بسته به اينكه جلوتر يا عقب تر از ماكسيمم هست ميتونيد مساحت رو با مثلث يا مستطيل يا هر چي به نظرتون بهتر فيت ميشه تقريب بزنين.
شار يه طول موج خاص رو هم اگه بخواين به جاي lambda تو رابطه مقدار طول موجو ميدين يا تو نمودارش مقدار متناظرش رو پيدا مي كنيد.

ali s.k
11-11-2013, 11:17 PM
ضریب 2 در این فرمول برای چیه؟ λ/λ=2rω/c∆
این فرمول برای به دست اوردن دوره ی تناوب چرخشی سیاره (چرخش به دور خود) به وسیله ی رادار است.در این روش یک موج راداری به یکی از لبه های استوایی سیاره می فرستند و موج پس از برخورد با لبه منعکس شده و به ناظر زمینی می رسد.در واقع این فرمول رابطه ای بین انتقال دوپلری و دوره تناوب چرخشی سیاره است.در این فرمول λ طول موج اولیه,λ∆ تغییرات طول موج,r شعاع استوایی,ω سرعت زاویه ای سیاره حول محور چرخش خود و c سرعت نور می باشد.
با تشکر

ali zeynali
11-12-2013, 10:22 AM
ضریب 2 در این فرمول برای چیه؟ λ/λ=2rω/c∆
این فرمول برای به دست اوردن دوره ی تناوب چرخشی سیاره (چرخش به دور خود) به وسیله ی رادار است.در این روش یک موج راداری به یکی از لبه های استوایی سیاره می فرستند و موج پس از برخورد با لبه منعکس شده و به ناظر زمینی می رسد.در واقع این فرمول رابطه ای بین انتقال دوپلری و دوره تناوب چرخشی سیاره است.در این فرمول λ طول موج اولیه,λ∆ تغییرات طول موج,r شعاع استوایی,ω سرعت زاویه ای سیاره حول محور چرخش خود و c سرعت نور می باشد.
با تشکر
ضريب ٢ براي اينه كه λ∆ اختلاف طول موج بينه فوتوني كه از لبه دور شونده مياد با فوتوني كه از لبه نزديك شونده مياد هستش واين اختلاف دو برابر اختلاف بين يكي از اون فوتون ها با فوتوني كه از مركز مياد هستش

ali s.k
11-12-2013, 06:08 PM
ضريب ٢ براي اينه كه λ∆ اختلاف طول موج بينه فوتوني كه از لبه دور شونده مياد با فوتوني كه از لبه نزديك شونده مياد هستش واين اختلاف دو برابر اختلاف بين يكي از اون فوتون ها با فوتوني كه از مركز مياد هستش
چگونه می توان این موضوع رو اثبات کرد؟

maryam hashemi
11-12-2013, 07:52 PM
سلام.يه سوال مكانيك هست.ميشه راه حلشونو كامل بگين؟
1.ماهواره اي را از روي استوا مماس بر زمين با چه سرعتي بفرستيم تا خروج از مركز مدارش 0.3باشد؟
مشكا اينجاست كه Reوrوaرو نمي تونم بفهمم چه جوري بايد تو فرمول انرژي جايگذاري كنم!

ali s.k
11-15-2013, 11:33 AM
بازم سلام!
ببينيد، نمودار پلانك يك نمودار توزيع و تابعش هم يك تابع توزيع هست. تابع توزيع يعني از هر كميتي(x) چند تا(f(x داريم
براي اينكه بهتر متوجه بشين يه مثال ميزنم: فرض كنيد تابع توزيع جرم يك خوشه ستاره رو داده باشن، يعني بگن از هر
جرمي توي اون خوشه چند تا هست. حالا قبول دارين كه اگر جرم هر ستاره رو ضرب در تعدادش كنيم بعد با بعدي جمع بزنيم همينجور تا اخر جرم كل خوشه رو ميده؟ حالا اگه حد اين سيگما رو به سمت بي نهايت و مقدار جرم رو به بازه هاي كوچك ( ديفرانسيلي) تقسيم كنيم ميشه انتگرال اون تابع از ٠ تا بي نهايت . توي نمودار پلانك هم عملا شار در هر طول- موجو تو طول موجش ضرب مي كنيم. كه اينم مثل قبلي ميشه انتگرال از ٠ تا بي نهايت (.B(lam.)d(lam . حالا اگه انتگرال رو تو محدوده خاص بخواين بگيرين ، مثلا ٤٥٠ تا ٥٥٠ نانو متر دو راه هست ؛ اولي اينكه انتگرال رو تو اون بازه بگيريد
دومي اينكه اگر مثل الان بازه زياد بزرگ نبود، بسته به اينكه جلوتر يا عقب تر از ماكسيمم هست ميتونيد مساحت رو با مثلث يا مستطيل يا هر چي به نظرتون بهتر فيت ميشه تقريب بزنين.
شار يه طول موج خاص رو هم اگه بخواين به جاي lambda تو رابطه مقدار طول موجو ميدين يا تو نمودارش مقدار متناظرش رو پيدا مي كنيد.
سلام
با تشکر از پاسختون
در نمودار پلانک شدت در هر طول موج رو باید در طول موجش ضرب کنیم چون نمودار پلانک نمودار شدت-طول موج هست. اما با توجه به تعریف شدت چگونه مجموع مساحت های این مستطیل ها برابر شار می شود؟

soorena
11-15-2013, 06:59 PM
سلام.يه سوال مكانيك هست.ميشه راه حلشونو كامل بگين؟
1.ماهواره اي را از روي استوا مماس بر زمين با چه سرعتي بفرستيم تا خروج از مركز مدارش 0.3باشد؟
مشكا اينجاست كه Reوrوaرو نمي تونم بفهمم چه جوري بايد تو فرمول انرژي جايگذاري كنم!

سلام يه راه حل أوليه كه برأي أين سؤال به ذهنم ميرسه اينه كه مدار ماهواره رو بيضي به مركز زمين در نظر مي گيريم با توجه به أين كه وقتي ماهواره روي زمينه فاصله إز كانون برابر Re است ، رابطه ي انرژي رو مينويسيم إز طرفي چون ماهواره رو مماس با زمين و إز روي استوا مي فرستيم زاويه بين بردار سرعت و بردار فاصله٩٠ درجه است پس را بطه ي تكانه زاويه اي را هم مينويسيم و aرا كه بر حسب سرعت إز رابطه ي انرژي به دست آورده ايم در رابطه تكانه جايگزين مى كنيم حال معادله اي فقط بر حسب مجهول v ( سرعت ) داريم.

celestial boy
11-15-2013, 07:12 PM
سلام.يه سوال مكانيك هست.ميشه راه حلشونو كامل بگين؟
1.ماهواره اي را از روي استوا مماس بر زمين با چه سرعتي بفرستيم تا خروج از مركز مدارش 0.3باشد؟
مشكا اينجاست كه Reوrوaرو نمي تونم بفهمم چه جوري بايد تو فرمول انرژي جايگذاري كنم!

سلام
خب شما میتونید h تکانه ویژه مدار رو بر حسب سرعت اولیه (که مجهوله) بنویسید (مماس بر زمین هم هست.پس زاویه مسیر هم داریم)
انرژی مدار رو هم میتونیم بر حسب همون سرعت اولیه بنویسیم.(شعاع مدار در اون نقطه شعاع زمینه). بعد نیم قطر a از انرژی مدار به دست میاد.حالا میذارید توی فرمول خروج از مرکز و سرعت رو ازش حساب می کنید.
به طور خیلی خلاصه:

h=R.V­0­­
E=1/2 V­­­0­­­­­^2 - GM/R
a=-GM/(2E)l
a(1-e^2)=h^2 /(GM)l
?=V0
(اگه بد شده ببخشید)

--------------------------------------
(من پست سورنا رو ندیده بودم!)

soorena
11-15-2013, 07:18 PM
سلام يه راه حل أوليه كه برأي أين سؤال به ذهنم ميرسه اينه كه مدار ماهواره رو بيضي به مركز زمين در نظر مي گيريم با توجه به أين كه وقتي ماهواره روي زمينه فاصله إز كانون برابر Re است ، رابطه ي انرژي رو مينويسيم إز طرفي چون ماهواره رو مماس با زمين و إز روي استوا مي فرستيم زاويه بين بردار سرعت و بردار فاصله٩٠ درجه است پس را بطه ي تكانه زاويه اي را هم مينويسيم و aرا كه بر حسب سرعت إز رابطه ي انرژي به دست آورده ايم در رابطه تكانه جايگزين مى كنيم حال معادله اي فقط بر حسب مجهول v ( سرعت ) داريم.
البته الآن كه فكر مي كنم يه راه حل ساده ترم اينه كه ابتدأ مثل دفعه قبل انرژي رو مينويسيم چون كانون مركز زمينه و ماهواره روي استواست پس زاويه تتا برابر ٩٠ درجست و فاصله هم برابر Re مقدار a رو إز رابطه قطبي بيضي به دست مي آوريم و داخل معادله انرژي جايگزين مي كنيم.(البته با عرض پوزش منم راه حل استاد حسن پور رو نديده بودم)

veno0s
12-01-2013, 06:48 PM
سلام

1-اگرrشعاع زمین باشددرچه فاصله ای ازسطح زمین برحسب r نیروی گرانش یک چهارم مقداران درسطح زمین است؟


2-جرم کره ی زمین تقریبا80برابرجرم کره ی ماه است وشتاب گرانشی درسطح زمین تقریبا6برابرشتاب گرانشی درسطح ماه است شعاع زمین تقریباچندبرابرشعاع ماه است؟

celestial boy
12-02-2013, 10:21 AM
سلام

1-اگرrشعاع زمین باشددرچه فاصله ای ازسطح زمین برحسب r نیروی گرانش یک چهارم مقداران درسطح زمین است؟


2-جرم کره ی زمین تقریبا80برابرجرم کره ی ماه است وشتاب گرانشی درسطح زمین تقریبا6برابرشتاب گرانشی درسطح ماه است شعاع زمین تقریباچندبرابرشعاع ماه است؟

سلام.
سوال اول: اگه فرمول شتاب گرانش رو بلد باشید (GM/r^2) می تونید بنویسید شتاب در سطح زمین مساوی با یک چهارم شتاب در فاصله r از مرکز زمین(دقیقا متن سوال رو به ریاضی ترجمه کنید!).GM ها از دو طرف حذف میشن.و r به دست میاد.حالا r-R (اختلاف r به دست اومده و R که شعاع زمینه) میشه جواب مسئله.

سوال دوم: باز هم از همون فرمول نسبت شتاب ها در سطح زمین و سطح ماه رو بنویسید.اینبار M ها خط نمی خورن.یه ضرب و تقسیم ساده و در نهایت جواب.

mjaasgari
01-15-2014, 05:05 PM
سلام علیکم
سوال:کمربند کوییپری چیه؟
سوال2:برای سوالی که در مرحله 2 پارسال در مورد کمربند کوییپری اومده بود یه ایده بدید!(راهنمایی)
ممنون

Amir Asadzadeh
01-15-2014, 09:07 PM
سلام علیکم
سوال:کمربند کوییپری چیه؟
سوال2:برای سوالی که در مرحله 2 پارسال در مورد کمربند کوییپری اومده بود یه ایده بدید!(راهنمایی)
ممنون
کمربند کوییپر (Kuiper belt) منطقه‌ای از منظومه شمسی دورتر از سیارات است که از مدار نپتون (فاصله 30 واحد نجومی) آغاز می‌شود و تا فاصله حدود 50 واحد نجومی از خورشید ادامه دارد.
این کمربند شبیه کمربند سیارک‌هاست البته بسیار بزرگتر است: 20 برابر پهن‌تر و 20 تا 200 برابر پر جرم‌تر. کمربند کوییپر مانند کمربند سیارک‌ها عمدتا شامل تکه‌های کوچک یا باقی‌مانده از تشکیل منظومه شمسیاست. در حالی که کمربند سیارک‌ها بیشتر از سنگ، یخ و فلز تشکیل شده است، کمربند کوییپر عمدتا حاوی مواد یخ زده مانند متان، آمونیاک و آب است.

برای جزییات بیشتر به لینک زیر مراجعه کنید:
http://wiki.avastarco.com/index.php?title=%DA%A9%D9%85%D8%B1%D8%A8%D9%86%D8% AF_%DA%A9%D9%88%DB%8C%DB%8C%D9%BE%D8%B1
این سوال یکی از چالش برانگیز ترین سوالای مرحله 2 تابحال بوده است چون هیچ کس (حتی طلایی های سال پیش) نمره ی کامل از این سوال نگرفت!! :D
در این سوال در قسمت (الف) باید سرعت جسم کوییپری (که در مداری دایروی به دور خورشید حرکت می کند) را بدست می آوریم و با دانستن فاصله ی آن از ناظر سرعت زاویه ای اش را می یابیم.
ولی در قسمت (ب) باید از دید اختلاف منظری به قضیه نگاه کنیم یعنی محاسبه کنیم که در طی نیم سال این سیارک ( که در این قسمت آن را ثابت می پنداریم) از دید ناظر زمینی چه زاویه ای را در آسمان طی کرده و با دانستن زمان (نیم سال) سرعت زاویه ای را بدست می آوریم.
در قسمت (ج) نیز باید سرعت نسبی سیارک را نسبت به زمین بدست آوریم و ... ( اگه دیگه این رو هم میگفتم سوالو حل میکردم در حالی که میخواستم راهنمایی کنم!!) :)

fatemeh.zar
01-20-2014, 10:47 AM
یک سوالی که نتونستم حل کنم:

اگر فرض کنیم ناگهان تمام واکنش های هسته ای و انرژی و فشار در خورشید متوقف بشه، چقدر طول میکشه تا خورشید در یک نقطه جمع بشه؟؟؟

پ.ن: کسی میتونه این معادله دیفرانسیلی رو حل کنه؟ ایکس* ایکس دبل دات= ثابت
:17:

Hosein Hashemi
01-20-2014, 01:31 PM
یک سوالی که نتونستم حل کنم:

اگر فرض کنیم ناگهان تمام واکنش های هسته ای و انرژی و فشار در خورشید متوقف بشه، چقدر طول میکشه تا خورشید در یک نقطه جمع بشه؟؟؟

پ.ن: کسی میتونه این معادله دیفرانسیلی رو حل کنه؟ ایکس* ایکس دبل دات= ثابت
:17:
نظر من اینه :
به عنوان یه تقریب فرض می کنیم درخشندگی خورشید ثابت بمونه .
بعدش تمام درخشندگی شو باید انقباض گرانشی ش جبران کنه !
بنابراین تغییرات انرژی گرانشی ش از لحظه ی شروع انقباض تا جمع شدن در یه نقطه تقسیم بر زمان انقباض برابر میشه با درخشندگی خورشید .
که از این رابطه زمانش به دست میاد ...
پ . ن : معادله دیفرانسیل ت با ۲ بار انتگرال گیری درمیاد ! ایکس دابل دات رو بنویس وی دات ! بعدش دو طرف رو در dx ضرب کن ! یه طرف یه انتگرال آسون برا v داری
یه طرف دیگه هم انتگرال dx/x که میشه Lnx . بعدش v رو هم می نویسی dx/dt و خلاص ... اگر لازمه بگو تا بنویسمش ;)

fatemeh.zar
01-20-2014, 03:23 PM
نظر من اینه :
به عنوان یه تقریب فرض می کنیم درخشندگی خورشید ثابت بمونه .
بعدش تمام درخشندگی شو باید انقباض گرانشی ش جبران کنه !
بنابراین تغییرات انرژی گرانشی ش از لحظه ی شروع انقباض تا جمع شدن در یه نقطه تقسیم بر زمان انقباض برابر میشه با درخشندگی خورشید .
که از این رابطه زمانش به دست میاد ...
پ . ن : معادله دیفرانسیل ت با ۲ بار انتگرال گیری درمیاد ! ایکس دابل دات رو بنویس وی دات ! بعدش دو طرف رو در dx ضرب کن ! یه طرف یه انتگرال آسون برا v داری
یه طرف دیگه هم انتگرال dx/x که میشه Lnx . بعدش v رو هم می نویسی dx/dt و خلاص ... اگر لازمه بگو تا بنویسمش ;)

فکر میکنم کتاب مادرن هم همینکارو کرده. راه دیگه ای نداره؟؟؟ مثلا اگه شرایط یه جوری باشه که درخشندگی رو ندونیم؟

http://up.avastarco.com/images/vy4mbk6fxaxh0zwwmm0l.gif (http://up.avastarco.com/)

بعدش چی؟؟؟
بعد اگه بخوایم ثابت های انتگرال رو هم بذاریم خ سخت میشه؟؟:oh noes:

پ.ن: ببخشید عکس یکم تو هم شد آخه هرکاری کردم نرفت سر خط!

mjaasgari
01-20-2014, 05:03 PM
سلام یه سوالی برام پیش اومده؟(البته دوتا:D)
1.اگه همین الان خورشید رو از منظومه شمسی حذف کنیم کی توجه میشیم که خورشید نیست؟(از رو ی زمین) با نور که تقریبا 8 دقیقه ولی من ایو میخواستم که آیا گرانش هم سرعتی معادل سرعت نور داره؟آیا نسبیت عام این موضوع رو تایید میکنه؟یا اینکه ما از روی گرانشی که از بین میرود میتوان بلافاصله به تابود شدن خورشید پی برد؟
2.اگه همین الان ماه رو از وسط به دوقسمت مساوی تقسیم کنیم چه اتفاقی میافته؟در 3 حالت که فقط نصف شود و دیگری با نصف شدندش نیرویی به ان وارد شود تا اینکه این دو قسمت بدلیل جاذبه گرانشی دوباره با هم برخورد نکنند و همچنین این نیرو به قدری باشد که قسمت ها از هم در حال دور شدن باشند و در فاصله d نسبت به هم قرار نگیرند (طبق فزض قسمت 2)

Hosein Hashemi
01-20-2014, 08:36 PM
فکر میکنم کتاب مادرن هم همینکارو کرده. راه دیگه ای نداره؟؟؟ مثلا اگه شرایط یه جوری باشه که درخشندگی رو ندونیم؟

http://up.avastarco.com/images/vy4mbk6fxaxh0zwwmm0l.gif (http://up.avastarco.com/)

بعدش چی؟؟؟
بعد اگه بخوایم ثابت های انتگرال رو هم بذاریم خ سخت میشه؟؟:oh noes:

پ.ن: ببخشید عکس یکم تو هم شد آخه هرکاری کردم نرفت سر خط!
نه خب ببین با انقباض گرانشی داره انرژی آزاد میشه ! بعد پایستگی انرژی حکم می کنه که این انرژی آزاد شده به یه چیزی تبدیل شه که میگیم به تابش تبدیل میشه !
بنابراین اگه ما چیزی از درخشندگی ندونیم در واقع معادله مون ناقصه ! میگیم یه انرژی گرانشی ای داره آزاد میشه ولی نمی دونیم برابر با چیه که از توش زمان رو در بیاریم !
بنابراین باید یه اطلاعاتی از درخشندگی داشته باشیم ! باید :دی
انتگرال سمت راستی که یه چیز واضحی هست ! می مونه انتگرال سمت چپی که یا باید بری در مورد روش های انتگرال گیری بخونی تا بتونی حلش کنی . یا می تونی بری
تو جدول انتگرال گیری نگاه کنی انتگرالش چی میشه :دی با گذاشتن ثابت ها صرفا یه مقدار محاسباتت طولانی تر میشه ! چیزی سخت نمیشه ! ;)

Hosein Hashemi
01-20-2014, 08:43 PM
سلام یه سوالی برام پیش اومده؟(البته دوتا:D)
1.اگه همین الان خورشید رو از منظومه شمسی حذف کنیم کی توجه میشیم که خورشید نیست؟(از رو ی زمین) با نور که تقریبا 8 دقیقه ولی من ایو میخواستم که آیا گرانش هم سرعتی معادل سرعت نور داره؟آیا نسبیت عام این موضوع رو تایید میکنه؟یا اینکه ما از روی گرانشی که از بین میرود میتوان بلافاصله به تابود شدن خورشید پی برد؟
2.اگه همین الان ماه رو از وسط به دوقسمت مساوی تقسیم کنیم چه اتفاقی میافته؟در 3 حالت که فقط نصف شود و دیگری با نصف شدندش نیرویی به ان وارد شود تا اینکه این دو قسمت بدلیل جاذبه گرانشی دوباره با هم برخورد نکنند و همچنین این نیرو به قدری باشد که قسمت ها از هم در حال دور شدن باشند و در فاصله d نسبت به هم قرار نگیرند (طبق فزض قسمت 2)
صرفا نظر شخصی مه و دلیلی بر درست بودنش نیست :4: :
۱. تا اونجا که من می دونم همون طور که نور از ذراتی به نام فوتون تشکیل شده ، امواج گرانشی هم از ذراتی به نام گراویتون تشکیل شدن . این ذرات رو فیزیکدان ها برای توجیه گرانش در نظر میگیرن و میگن
سرعتشون همون سرعت نوره ! مطمئن نیستم باید تو منابع علمی معتبر چک ش کنم !
۲. ۳ حالتشو متوجه نشدم راستش :دی ینی چی فقط نصف شن ؟

mjaasgari
01-21-2014, 04:54 AM
صرفا نظر شخصی مه و دلیلی بر درست بودنش نیست :4: :
۱. تا اونجا که من می دونم همون طور که نور از ذراتی به نام فوتون تشکیل شده ، امواج گرانشی هم از ذراتی به نام گراویتون تشکیل شدن . این ذرات رو فیزیکدان ها برای توجیه گرانش در نظر میگیرن و میگن
سرعتشون همون سرعت نوره ! مطمئن نیستم باید تو منابع علمی معتبر چک ش کنم !
۲. ۳ حالتشو متوجه نشدم راستش :دی ینی چی فقط نصف شن ؟

خوب بابا وقتي ماه دوتا تكه بشه اين دوتا تيكه سه حالت نصبت به هم دارن يا در فاصله ثابت از هم قرار ميگيرند يا همش در حال دور شدن اند يا همون جا بدليل گرانش دوباره به هم ميچسبند

fatemeh.zar
01-21-2014, 04:01 PM
نه خب ببین با انقباض گرانشی داره انرژی آزاد میشه ! بعد پایستگی انرژی حکم می کنه که این انرژی آزاد شده به یه چیزی تبدیل شه که میگیم به تابش تبدیل میشه !
بنابراین اگه ما چیزی از درخشندگی ندونیم در واقع معادله مون ناقصه ! میگیم یه انرژی گرانشی ای داره آزاد میشه ولی نمی دونیم برابر با چیه که از توش زمان رو در بیاریم !
بنابراین باید یه اطلاعاتی از درخشندگی داشته باشیم ! باید :دی


نمیشه قانون دوم نیوتن رو برای یه جزء دیفرانسیلی جرم روی سطح ستاره بنویسیم و حل کنیم؟؟؟

در واقع همون معادله دیفرانسیلی عجیب رو می داد که من بار اول یه دات کم گذاشته بودم(!) خ راحت حل شده بود! کلللی هم ذوق کردم:دی

veno0s
01-22-2014, 10:49 PM
سلام لطفاقانون وین روهمراه بافرمولش توضیح بدید

واین سوال که سوال12مرحله اول پارسال هست:

طول موج تابش زمینه ی کیهان برحسب متربه کدام گزینه نزدیکتراست؟

1)2/7ضربدر10به توان3 2)2/7 3)5/3در10به توان منفی3 4)5/3در10به توان منفی6

شروین حکیمی
01-23-2014, 12:41 AM
سلام لطفاقانون وین روهمراه بافرمولش توضیح بدید

واین سوال که سوال12مرحله اول پارسال هست:

طول موج تابش زمینه ی کیهان برحسب متربه کدام گزینه نزدیکتراست؟

1)2/7ضربدر10به توان3 2)2/7 3)5/3در10به توان منفی3 4)5/3در10به توان منفی6



معادله ی تابش پلانک را در نظر بگیرید: I(λ)dλ=(2hc^2)/λ^5 ×1/(e^(hc/λkt)-1) dλ
این معاداه نشان می دهد که برای هر جسم سیاهی در طول موج های مختلف، شدت مختلفی را دریافت می کنیم؛ اگر نمودار آن را در نظر بگیریم:
http://upload.wikimedia.org/wikipedia/commons/a/a2/Wiens_law.svg

( یک نکته در مورد این نمودار ، در دما های بیشتر در همه ی طول موج ها شدتی بیشتری را دریافت می کنیم)

برای هر دمایی یک طول موجی هست که بیشترین شدت را دریافت می کنیم، این طول موج را می توان با مشتق گرفتن تابع پلانک برحسب طول موج بدست آورد؛ ( البته معادله ی آخرش را باید به ماشین حساب داد)
λ_max T=2.898×〖10〗^(-3)

دمای پس زمینه کیهان هم برابر با 2.75 کلوین هست پس نزدیک ترین گزینه، گزینه 3 درست هست.

tina
01-28-2014, 09:56 PM
سلام
چند تا سوال :
1- بیشترین سرعت شعاعی ماهواره تو مدار بیضوی وقتیه که r=a ؟!
2- اگه بیشترین سرعت شعاعی ماهواره و نیم قطر اطول مدارشو داشته باشیم خروج از مرکز رو چه جوری حساب کنیم ؟ !
3- برا نیرو های F=Kr^n زاویه اوج و حضیضش پی به روی رادیکال سه به علاوه ی n میشه (؟) بعد توی سوالای آقای چرتاب زده بود شرط اینکه مدار بسته باشه اینه که رادیکال سه به علاوه ی اِن صحیح باشه . .. مدار بسته یعنی چی ؟ ! من فکر میکردم یه چیز مثه سهمی بسته نیست ولی اگه این شرط بر قرار باشه کلا نیروی کا به روی آر دو باید مدارش بسته باشه :-?

m.Sadat
01-28-2014, 11:04 PM
سلام
چند تا سوال :
1- بیشترین سرعت شعاعی ماهواره تو مدار بیضوی وقتیه که r=a ؟!
2- اگه بیشترین سرعت شعاعی ماهواره و نیم قطر اطول مدارشو داشته باشیم خروج از مرکز رو چه جوری حساب کنیم ؟ !

از زاویه بین راستای مماسی و بردار سرعت یا همون گاما استفاده کنید

Amir Asadzadeh
01-29-2014, 12:14 AM
سلام
چند تا سوال :
1- بیشترین سرعت شعاعی ماهواره تو مدار بیضوی وقتیه که r=a ؟!
2- اگه بیشترین سرعت شعاعی ماهواره و نیم قطر اطول مدارشو داشته باشیم خروج از مرکز رو چه جوری حساب کنیم ؟ !
تو فرمول های زیر h همون تکانه زاویه ای به واحد جرم تو بیضیه.
6254
فک کنم جواب دو تا سوال رو داده باشم.
از آخرین فرمول در صورتی که M و a و سرعت شعاعی ماکسیمم رو داشته باشیم می تونیم خروج از مرکز رو به راحتی حساب کنیم. :have%20a%20nice%20d

tina
01-29-2014, 10:12 AM
سلام :دی مرسی که جواب دادین یه سوال دیگه :
چه جوری مساحت مشترک دو تا عرقچین رو حساب کنم ؟ (مثه سوال برج یانقین آقای صدرالدینی)

celestial boy
01-29-2014, 02:19 PM
سلام :دی مرسی که جواب دادین یه سوال دیگه :
چه جوری مساحت مشترک دو تا عرقچین رو حساب کنم ؟ (مثه سوال برج یانقین آقای صدرالدینی)

سلام.
این یه روش حل از آقای عطا مرادی.
(http://up.avastarco.com/images/wlpdknw9dawbs9alel1m.pdf)

mjaasgari
02-04-2014, 10:02 AM
سلام
سه تا سوال
آنومالي چيه تعريف توضيح فرمول روش بدت آوردن.؟
حد روش يا روچ چيه تعريف توضيح روش بدت آوردن ؟
شعاع شوارتز شيلد چيه تعريف..........................؟
ممنون

celestial boy
02-04-2014, 11:49 AM
سلام
سه تا سوال
آنومالي چيه تعريف توضيح فرمول روش بدت آوردن.؟
حد روش يا روچ چيه تعريف توضيح روش بدت آوردن ؟
شعاع شوارتز شيلد چيه تعريف..........................؟
ممنون

عجب سوال کوچیکی.جوابشم زیاد نیست.در حد دو سه تا کتاب :دی
آنومالی: خیلی ریشه ای و اساسی توی این تاپیک (http://forum.avastarco.com/forum/showthread.php?792-%D9%85%DA%A9%D8%A7%D9%86%DB%8C%DA%A9-%D9%85%D8%AF%D8%A7%D8%B1%DB%8C) آنومالی (ها) تعریف شدن.
حد روچ: وقتی قمر یه سیاره (یا هر جسمی) به سیاره نزدیک بشه٬ یه حدی برای نزدیک شدنش وجود داره.یعنی اگه از یه فاصله ای نزدیک تر بشه متلاشی میشه.این حد به مشخصات قمر و سیاره مربوطه.مثلا قمر سیال (گازی) و قمر صلب دو حد متفاوت دارن.دلیل اصلی متلاشی شدن٬ گرانش جزئی سیاره روی قمره (مثه همون پدیده جذر و مد.اینبار خشن تر)
حد روچ توی منظومه های ستاره ای هم هست.اما یه سری تفاوت هایی داره.توی این منظومه ها وقتی یه ستاره به غول تبدیل میشه٬ ممکنه اونقد بزرگ بشه که سطحش از حد روچ خارج بشه.در این حالت گاز ها از غول به سمت ستاره دیگه سرازیر میشن.
محاسباتشو دیگه شرمنده.برید کتابا رو نگاه کنید. :دی
شعاع شوارتزشیلد که شعاع سیاهچاله هم بهش میگن مقداریه که اگه نور ازون بیشتر به سیاهچاله نزدیک بشه نمیتونه ازش فرار کنه.کافیه توی فرمول سرعت فرار٬ سرعت فرار رو برابر سرعت نور قرار بدید.شعاعی که به دست میاد شعاع شوارتزشیلد خواهد بود.اینم توضیح مبسوط تر (http://fa.wikipedia.org/wiki/%D8%B4%D8%B9%D8%A7%D8%B9_%D8%B4%D9%88%D8%A7%D8%B1% D8%AA%D8%B2%D8%B4%DB%8C%D9%84%D8%AF)

tina
02-06-2014, 08:55 PM
سلام :دی
یه سواله میگه دو تا ماهواره داریم که توی مدار دایروی همسان ساعتگرد دور زمین میچرخن( با ارتفاع 10000 کیلومتر از سطح زمین ) از دید ناظر زمینی هم 50 درجه اختلاف زاویه ای دارن جرم هاشون رو هم داده حالا میگه میخوایم با حداقل انرژی لازم کاری کنیم که این دو تا ماهواره بهم بخورن( بعد توی صورت سوال هم نگفته که کدوم جلو تره) .
اگه نگفته بود حداقل انرژی میگفتیم خوب یه مقدار انرژی میدیم یا میگیریم که سرعتش عوض شه بیفته تو مدار بیضوی که بعد از یه دور چرخیدن یا دو دور چرخیدن یا هر چند دور چرخیدن اون ماهواره بیفته تو اوج یا حضیض بیضی و اینا هم زمان اون جا باشن بخورن به هم ولی اینکه گفته حداقل انرژی یکم(خیلی !) گیجم کرده
خوب ما یه اپسیلون انرژی هم از ماهواره بگیریم یا به ماهواره بدیم بالاخره یه روزی این دو تا ماهواره هم زمان تو یه نقطه هستن و با هم برخورد میکنن ، مگه نه ؟!

tina
02-24-2014, 04:46 PM
جواب سوال قبلی رو خودم فهمیدم ...
گویا حالت بهینه یعنی هم از نظر انرژی هم زمان .
یه سوال دیگه (امیدوارم جواب بدین )
تو سیستم های دوتایی وقتی انتقال جرم صورت میگیره باعث میشه حتی وقتی گرفت ندارن هم روشناییشون تغییر کنه ؟ بعد اگه منحنی روشنایی بر حسب زمانشو بکشیم ببینیم داره وقتی گرفت نداره هم تغییر میکنه یعنی انتقال جرم داریم ؟ یا علت دیگه ای داره ؟ ! (چه جمله ای !! :| )
مرسی

ali zeynali
02-24-2014, 05:54 PM
جواب سوال قبلی رو خودم فهمیدم ...
گویا حالت بهینه یعنی هم از نظر انرژی هم زمان .
یه سوال دیگه (امیدوارم جواب بدین )
تو سیستم های دوتایی وقتی انتقال جرم صورت میگیره باعث میشه حتی وقتی گرفت ندارن هم روشناییشون تغییر کنه ؟ بعد اگه منحنی روشنایی بر حسب زمانشو بکشیم ببینیم داره وقتی گرفت نداره هم تغییر میکنه یعنی انتقال جرم داریم ؟ یا علت دیگه ای داره ؟ ! (چه جمله ای !! :| )
مرسی
کلا درخشندگی که برای دیسک های برافزایشی تامین میشه از ستاره ها تامین شده دیگه و فکر کنم مجموع درخشندگی دوستاره و دیسک با درخشندگی دو ستاره قبل انتقال فرق کنه. کلا انتقال جرم های بین دوستاره رو دو جور در نظر می گیرن. بعضی جا ها از زمان انتقال صرف نظر می کنند و محاسبات مداری بعد از انتقال رو می خوان. بعضی جا ها کلا انتقال رو طولانی در نظر می گیرند و درخشندگی و ... دیسک ناشی از براقزایش ماده رو بررسی می کنند.

mohammadmehdi
02-24-2014, 11:19 PM
توی تابع پلانک e چی هست؟

ali zeynali
02-25-2014, 09:50 AM
توی تابع پلانک e چی هست؟
e يك عدد ثابت هستش كه به آن عدد نپر مي گويند و حدود ٢.٧١ مقدار دارد

tina
02-25-2014, 02:00 PM
سلام
صفحه ی آسمان یه صفحه ی خاصه ؟ یا بستگی به ناظر داره ؟ مفهومی که از صفحه آسمون تو ذهن منه اینه که صفحه ی آسمون ناظر قطب جنوب به صفحه ی آسمان ناظر استوا عموده
بعد اگه این جوری باشه تو هر زمان از سال هم این صفحه فرق داره ...بعد من الان به یه سوالی برخورد کردم که میگه سیاره ی داخلی ای با طول حضیض 100... خوب یعنی این سیاره هه زمین هر جایی باشه ناظر هر جایی باشه طول حضیضش صده ؟ :-/ یا فقط برا این زمان و این ناظر اینو گفته ؟(من تو مفاهیم اولیه مشکل دارم :17: )

soroosh
02-25-2014, 08:11 PM
طول حضیض ربطی به ناظر و اینا نداره
طولی که اینجا میگه همون طول سماویه که جدایی زاویه ای حضیض مدار سیاره از اعتدال بهاری از دید خورشیده به صورت پاد ساعتگرد

ali zeynali
02-25-2014, 10:06 PM
سلام
صفحه ی آسمان یه صفحه ی خاصه ؟ یا بستگی به ناظر داره ؟ مفهومی که از صفحه آسمون تو ذهن منه اینه که صفحه ی آسمون ناظر قطب جنوب به صفحه ی آسمان ناظر استوا عموده
بعد اگه این جوری باشه تو هر زمان از سال هم این صفحه فرق داره ...بعد من الان به یه سوالی برخورد کردم که میگه سیاره ی داخلی ای با طول حضیض 100... خوب یعنی این سیاره هه زمین هر جایی باشه ناظر هر جایی باشه طول حضیضش صده ؟ :-/ یا فقط برا این زمان و این ناظر اینو گفته ؟(من تو مفاهیم اولیه مشکل دارم :17: )
بزاريد كمي درباره اينها توضيح بدم:
مدار زمين به دور خورشيد درون صفحه اي قرار داره كه به اون دايره البروج مي گويند. حال صفحه مداري هر جسمي كه به دور خورشيد مي گردد در صورت هم صفحه نبودن با دايره البروج در دو نقطه مدار زمين را قطع مي كند. به آن نقطه كه سياره در آن نقطه صعود مي كند نقطه گره صعودي مي گويند. حال به فاصله زاويه اي اين نقطه تا نقطه حضيض مدار طول حضيض يا شناسه حضيض مي گويند. دقت كنيد كه مركز اين زاويه خورشيد است.
مشكلتان در صفحه آسمان را متوجه نشدم ولي فكر كنم اوني كه شما مي گيد بز هم عموده ، افقه ناظران است نه صفحه آسمان. آسمان را كره اي در نظر مي گيرند و صفحه آسمان در هر نقطه تعريف مي شود. كه صفحه آسمان در هر نقطه از آسمان صفحه اي است كه بردار عمود بر آن بردار همان نقطه از ديد ناظري در مركز كره است.

tina
02-25-2014, 11:31 PM
آها
پس صفحه ی آسمان که برا دوتایی ها هست چیه ؟ که زاویه ی i رو از رو اون تعیین میکنن اون هم به ناظر بستگی نداره ؟ :blink:

شروین حکیمی
02-26-2014, 07:58 PM
شکل زیر فکر کنم کمک بکنه:

زاویه ی i زاویه ی تعریفش زاویه ی بین صفحه ی دید ما (NLD ) و صفحه ی مداری جسم (GLH ) هست و اگر ناظری مثلا در نقطه ی قطب (GLH) باشد i را 0 اندازه می گیرد.

http://8pic.ir/images/58526741613154311724.png (http://8pic.ir/)

ali zeynali
02-26-2014, 08:16 PM
آها
پس صفحه ی آسمان که برا دوتایی ها هست چیه ؟ که زاویه ی i رو از رو اون تعیین میکنن اون هم به ناظر بستگی نداره ؟ :blink:
نه . براي اينجور مساءل يك ناطر در نظر مي گيرند. ديگر زمين و كره زمين در مقياس هاي نجومي كوچك اند و غير مهم. ولي كمي فكر كنيد: اگر تعريف كره سماوي رو بفهميد كمك مي كنه. ما كره آسمان كه تعريف مي كنيم به منظور كره اي با شعاع خيلي خيلي خيلي زياد است. اگر اين شعاع خيلي بزرگ باشد مقياس زمين خيلي كوچك است پس دو ناظر كه حتي در دوسوي زمين قرار دارند در اين مقياس عملا روي مركز كره روي يك نقطه قرار دارند. يعني كره آسمان آنها يكي مي شود. فرض كنيد شما الان همينجا كه هستيد بتوانيد زير افق را ببينيد ، زير افق شما در واقع همان آسمان قابل رويت ناظر آنسوي زمين است.

fatemeh.zar
03-06-2014, 03:51 PM
سلام!
آقا لطفا یکی این مقر های تلسکوپ ها رو از اول تا آخر توضیح بده!!

tina
03-09-2014, 02:51 PM
سلام
تو این سوال نباید ضریب جذب و چگالی هوا رو میداد ؟
محاسبات ناظری در تبریز (فی =38)نشان میدهد ستاره ای در یک تاریخ خاص باید در طول شب دیده شود .و مدت زمانی که ستاره باید بالای افق باشد 13 ساعت و 40 دقیقه اندازه گیری میشود . اما به دلیل جذب تابش در جو زمین فقط به مدت یازده ساعت و ده دقیقه این ستاره با چشم غیر مسلح دیده میشود . اگر قدر این ستاره در روشن ترین وضعیت 4.6 اندازه گیری شود و قدر حدی چشم 6.5 باشد ارتفاع جو از سطح زمین چقدر است ؟

fatemeh.zar
03-09-2014, 07:06 PM
سلام
تو این سوال نباید ضریب جذب و چگالی هوا رو میداد ؟
محاسبات ناظری در تبریز (فی =38)نشان میدهد ستاره ای در یک تاریخ خاص باید در طول شب دیده شود .و مدت زمانی که ستاره باید بالای افق باشد 13 ساعت و 40 دقیقه اندازه گیری میشود . اما به دلیل جذب تابش در جو زمین فقط به مدت یازده ساعت و ده دقیقه این ستاره با چشم غیر مسلح دیده میشود . اگر قدر این ستاره در روشن ترین وضعیت 4.6 اندازه گیری شود و قدر حدی چشم 6.5 باشد ارتفاع جو از سطح زمین چقدر است ؟

سلام دوست عزیز

تو نسخه(!) ای که من از این سوال دارم گفته با این فرض که روشنایی یک ستاره با فاصله گذرنده رابطه ی عکس دارد، حساب کنید. :)

m.Sadat
03-09-2014, 08:26 PM
سلام
تو این سوال نباید ضریب جذب و چگالی هوا رو میداد ؟
محاسبات ناظری در تبریز (فی =38)نشان میدهد ستاره ای در یک تاریخ خاص باید در طول شب دیده شود .و مدت زمانی که ستاره باید بالای افق باشد 13 ساعت و 40 دقیقه اندازه گیری میشود . اما به دلیل جذب تابش در جو زمین فقط به مدت یازده ساعت و ده دقیقه این ستاره با چشم غیر مسلح دیده میشود . اگر قدر این ستاره در روشن ترین وضعیت 4.6 اندازه گیری شود و قدر حدی چشم 6.5 باشد ارتفاع جو از سطح زمین چقدر است ؟

در لینک زیر میتونید راه حل سوال رو ببینید
http://forum.avastarco.com/forum/showthread.php?767-%D8%B7%D8%B1%D8%AD-%D9%85%D8%B3%D8%A7%D8%A6%D9%84-%D8%A7%D9%84%D9%85%D9%BE%D9%8A%D8%A7%D8%AF%D9%8A&p=68359&viewfull=1#post68359

tina
03-17-2014, 10:30 AM
سلام
فرق شار و شدت چيه ؟ :دي

celestial boy
03-17-2014, 11:16 AM
سلام
فرق شار و شدت چيه ؟ :دي

سلام.
تا جایی که من می دونم و حضور ذهن دارم شدت مقدار انرژی هست که در واحد زمان در واحد زاویه فضایی در یه طول مرج خاص از واحد مساحت (مثلا یک متر مربع) می گذره.:)
مثلا یه جسم سیاه به صورت یه سوراخ در جعبه‌ دارید. اگر مقدار انرژی یک طول موج خاص تابش شده‌ی اون در واحد زمان (یا همون توان جسم سیاه در اون طول موج) رو تقسیم بر زاویه فضایی که داره منتشر میشه بکنید بعدش تقسیم بر مساحت اون سوارخ بکنید به این کمیت میگن شدت تابش.

اما اگه شما بیاید تمام شدت ها در طول موج های مختلف رو جمع بزنید و به زاویه فضایی هم تقسیم نکنید (شدت در همه زاویه فضایی ها رو جمع بزنید) این میشه شار.

یکم تعریفاشون گیج کنندس. از روی نمودار شدت بر حسب طول موج (مثه نمودار پلانک) بخوایم بگیم: محور عمودی همون شدت هست. برای هر طول موجی یه مقدار خاصی داره.
اما اگه مساحت زیر نمودار رو حساب کنیم یعنی شدت ها را روی تمام طول موج ها جمع زدیم. می مونه زاویه فضایی. اگه تابش در یه نیم کره باشه (مثه سطح ستاره) مساحت زیر نمودار ضرب در عدد پی میشه شار تابشی.یعنی روی زاویه فضایی جمع زدیم.

1997
03-17-2014, 12:10 PM
سلام
نزدیک مرحله دوم هستش.اگه میشه طلایی های عزیز به سوالات ما جواب بدند.
اینم از اولین سوال من:
ناظر a در شهری با عرض جغرافیایی 60 درجه شمالی و طول جغراقیایی 120غربی ماه را بر روی نصف النهار ناظر و با فاصله سمت الرسی 60 درجه رصد میکند . ناظر b در شهری با عرض جغافیایی 30 درجه جنوبی و طول جغرافیایی 120درجه غربی به طور همزمان با ناظر a ماه را بر روی نصف النهار ناظر و با فاصله سمت الراسی 31.28 رصد میکند. فاصله ماه تا مرکز زمین چند برابر شعاع زمین است ؟
من این رو اینگونه حل میکنم که از دو ناظر خطی از مرکز زمین به اونا می کشم بعد زاویه ی مرکز زمین و نقطه ای روی محیط زمین و ماه میشه 90+a که a برابر ارتفاع هستش که یه چهار ضلعی پدید میاد که نمی دونم چطوری باید حلش کنم.اگه میشه راهنماییم کنید.

celestial boy
03-17-2014, 03:53 PM
سلام
نزدیک مرحله دوم هستش.اگه میشه طلایی های عزیز به سوالات ما جواب بدند.
اینم از اولین سوال من:
ناظر a در شهری با عرض جغرافیایی 60 درجه شمالی و طول جغراقیایی 120غربی ماه را بر روی نصف النهار ناظر و با فاصله سمت الرسی 60 درجه رصد میکند . ناظر b در شهری با عرض جغافیایی 30 درجه جنوبی و طول جغرافیایی 120درجه غربی به طور همزمان با ناظر a ماه را بر روی نصف النهار ناظر و با فاصله سمت الراسی 31.28 رصد میکند. فاصله ماه تا مرکز زمین چند برابر شعاع زمین است ؟
من این رو اینگونه حل میکنم که از دو ناظر خطی از مرکز زمین به اونا می کشم بعد زاویه ی مرکز زمین و نقطه ای روی محیط زمین و ماه میشه 90+a که a برابر ارتفاع هستش که یه چهار ضلعی پدید میاد که نمی دونم چطوری باید حلش کنم.اگه میشه راهنماییم کنید.

سلام.
همه دوستان منتظرن بیایید و سوال بذارید. واقعا ناراحت میشه آدم وقتی میبینه تالار المپیاد آوااستار اینقد خلوت شده.

با این که اصلا دوست ندارم مستقیما جواب یه سوالو بذارم ولی ظاهرا خودتون ایده سوالو فهمیدید. این یه چند صفجه از کتاب اصول و عمل که عینا همین سوال شما رو حل کرده :)
قطر زاویه ای و فاصله سنجی (http://bayanbox.ir/id/2403726490336097101)

1997
03-17-2014, 05:17 PM
سلام
من این چیزی که شما گذاشتید رو خوندم ولی مشکل من حل عددی این سوال است.یعنی نمی دونم چطوری باید حلش کنم.اگه میشه راهنماییم کنید.
شکل سوال:http://s5.picofile.com/file/8117191318/DSC_0305.jpg

celestial boy
03-18-2014, 08:18 AM
سلام
من این چیزی که شما گذاشتید رو خوندم ولی مشکل من حل عددی این سوال است.یعنی نمی دونم چطوری باید حلش کنم.اگه میشه راهنماییم کنید.
شکل سوال:http://s5.picofile.com/file/8117191318/DSC_0305.jpg

من فک کردم توی اون چند صفحه راه حل رو نوشته. ظاهرا ننوشته.:)
من یه چیزی به ذهنم می رسه: خط واصل دو تا ناظر رو رسم کنید و اندازش رو به دست بیارید. بعد یه مثلث دیگه دارید بین ماه و دوتا ناظر؛ اونو حل کنید و فاصله ماه تا یکی از ناظرها رو به دست بیارید. بعد فاصله زمین-ماه به دست میاد.

یه نکته: توی شکلتون دقت کنید اون زاویه هایی که در مرکز زمین درست شدن برابر عرض ناظر ها نیستن. چون ماه لزوما روی استوا نیست. شما فقط می تونید از مجموع عرض ها استفاده کنید .

1997
03-18-2014, 11:09 AM
نمی دونم درست فهمیدم یا نه ولی فکر کنم در اون چهار ضلعی اون زاویه ی نزدیک زمین برابر 90 درجه هستش.
راستی یه نظری داشتم.برای فعال شدن این بخش بهتره هر روز یه نفر یه سوال بزاره مثل تورنمنت و هر کی ایده ی سوال قبل رو که گفت میریم سوال بعد.
برا شروع خودم اولین سوال رو میگم:
برای ناظری در عرض جغرافیایی فی حساب کنید در چه روزی از سال خورشید زودتر از بقیه ی روز ها طلوع می کند؟

1997
03-20-2014, 11:04 AM
حالا که مرحله اول تموم شد.
کسی نمی خواد به این سوال جواب بده.حالا که همش میاید تو تاپیک مرحله اول اعلام میکنید که قبول شدید بیاید این سوال رو حل کنید.
ای کسانی که قبول شدید در حل سوال مشارکت کنید.

tina
04-06-2014, 08:04 PM
سلام
توی این (http://up.avastarco.com/images/rq3ylxg61bs8y7j6h6x.jpg) سواله قسمت ب رو چه جوری باید حل کنم ؟ خواستم با قدر مطلق خورشید مقایسه کنم که سرعت چرخشش تو کهکشان خودمون رو هم داریم که اون ثابتی که در وی ضرب میشه بره ولی به یه مشکلی بر میخورم و اونم اینه که نمیشه قدر مطلق خورشید رو گذاشت اون ور رابطه چون در اون صورت قدر مطلق کهکشان یه چیز تو اردر قدر مطلق خورشید در میاد :|
بعد این (http://up.avastarco.com/images/kxlmyzvq3gdno67ou2n.jpg)فرمول رو هم میشه اثبات کنین ؟

fatemeh.zar
04-15-2014, 07:31 PM
سلام!
منم یه سوالی داشتم نمیدونم راهش چیه!
http://up.avastarco.com/images/q821bgi4nackjtwl5va2.jpg (http://up.avastarco.com/)

ali zeynali
04-15-2014, 08:02 PM
سلام!
منم یه سوالی داشتم نمیدونم راهش چیه!
http://up.avastarco.com/images/q821bgi4nackjtwl5va2.jpg (http://up.avastarco.com/)
خب از اینکه همزمان طلوع کنن میشه چی فهمید؟ درسته! میشه فهمید در هنگام طلوع هر دو, زمان نجومی یک مقدار است. این مقدار هم برابره با جمع زاویه ساعتی طلوع شعرا یمانی و بعد این ستاره که از داده های این مساله می تونید این مقدار رو بدست بیارید.
حالا برای خورشید هم این مقدار رو برابر با زاویه ساعتی طلوعش به علاوه بعدش بگذارید و بعد و زاویه ساعتی خورشید رو بر حسب میلش بنویسید.
حالا میل خورشید بدست میاد! وبعدش هم تاریخ.
البته یک داده کم این مساله عرض جغرافیایی اون شهرس که باید خودتون عددشو بزارید.

1997
04-16-2014, 08:51 AM
سلام
گفتم نزدیک مرحله دو هستش و همه در حال پرسیدن اشکالات خودشون هستن.
منم سوال داشتم.اگه میشه جواب بدید.فقط خداکنه سوال قبل نشه که هیچ کس جواب نداد.
http://up.avastarco.com/images/93szztahzv68bzt1xq2b.png

fatemeh.zar
04-16-2014, 03:57 PM
خب از اینکه همزمان طلوع کنن میشه چی فهمید؟ درسته! میشه فهمید در هنگام طلوع هر دو, زمان نجومی یک مقدار است. این مقدار هم برابره با جمع زاویه ساعتی طلوع شعرا یمانی و بعد این ستاره که از داده های این مساله می تونید این مقدار رو بدست بیارید.
حالا برای خورشید هم این مقدار رو برابر با زاویه ساعتی طلوعش به علاوه بعدش بگذارید و بعد و زاویه ساعتی خورشید رو بر حسب میلش بنویسید.
حالا میل خورشید بدست میاد! وبعدش هم تاریخ.
البته یک داده کم این مساله عرض جغرافیایی اون شهرس که باید خودتون عددشو بزارید.

خب همون دیگه! وقتی بعد و زاویه ساعتی رو میذاریم یه معادله ی عجیب معکوس مثلثاتی میشه که ماشین حساب هم نتونست حلش کنه!! این معادله:
http://up.avastarco.com/images/kskzk4165g2lb80k2lp.gif (http://up.avastarco.com/)
چجوری باید حلش کنم؟ (من فی رو 10 درجه در نظر گرفتم)

tina
04-16-2014, 09:51 PM
خب همون دیگه! وقتی بعد و زاویه ساعتی رو میذاریم یه معادله ی عجیب معکوس مثلثاتی میشه که ماشین حساب هم نتونست حلش کنه!! این معادله:
http://up.avastarco.com/images/kskzk4165g2lb80k2lp.gif (http://up.avastarco.com/)
چجوری باید حلش کنم؟ (من فی رو 10 درجه در نظر گرفتم)
سلام :دی
من راستش به این معادله ای که گفتی نرسیدم (البته به راه های من اعتمادی نیست یه مقداری داغون تشریف دارم :-" )
http://up.avastarco.com/images/v7f2e9buzgfkfginnl.jpg

starscream4002
04-17-2014, 05:46 AM
خب همون دیگه! وقتی بعد و زاویه ساعتی رو میذاریم یه معادله ی عجیب معکوس مثلثاتی میشه که ماشین حساب هم نتونست حلش کنه!! این معادله:
http://up.avastarco.com/images/kskzk4165g2lb80k2lp.gif (http://up.avastarco.com/)
چجوری باید حلش کنم؟ (من فی رو 10 درجه در نظر گرفتم)

به جای اینکه به طور مستقیم بخواین معادله رو حل کنین، اول یه رابطه مفید اثبات کنین تا با اون حل کنین. بهترین رابطه ممکن رابطه ی بین طول سماوی و st هست که به راحتی و فقط با چندتا جایگذاری به دست میاد.

بعد تو این رابطه به راحتی st رو جایگذاری میکنین و طول سماوی و در نتیجه تاریخ رو به دست میارین.
بازم اگه خواستین معادلات رو کامل می نویسم و اینجا به اشتراک می ذارم.

موفق باشید،

ss

fatemeh.zar
04-17-2014, 09:23 AM
به جای اینکه به طور مستقیم بخواین معادله رو حل کنین، اول یه رابطه مفید اثبات کنین تا با اون حل کنین. بهترین رابطه ممکن رابطه ی بین طول سماوی و st هست که به راحتی و فقط با چندتا جایگذاری به دست میاد.

بعد تو این رابطه به راحتی st رو جایگذاری میکنین و طول سماوی و در نتیجه تاریخ رو به دست میارین.
بازم اگه خواستین معادلات رو کامل می نویسم و اینجا به اشتراک می ذارم.

موفق باشید،

ss

ممنون!:) این رابطه میشه دیگه؟

http://up.avastarco.com/images/794bq7r4iikdswc3kidj.gif (http://up.avastarco.com/)

fatemeh.zar
04-17-2014, 09:49 AM
برا شروع خودم اولین سوال رو میگم:
برای ناظری در عرض جغرافیایی فی حساب کنید در چه روزی از سال خورشید زودتر از بقیه ی روز ها طلوع می کند؟

خب، من فکر میکنم که جواب مستقل از عرض جغرافیایی است و در نیمکره شمالی در روز انقلاب تابستانی و در نیمکره جنوبی در روز انقلاب زمستانی این اتفاق می افتد.


سلام
توی این (http://up.avastarco.com/images/rq3ylxg61bs8y7j6h6x.jpg) سواله قسمت ب رو چه جوری باید حل کنم ؟ خواستم با قدر مطلق خورشید مقایسه کنم که سرعت چرخشش تو کهکشان خودمون رو هم داریم که اون ثابتی که در وی ضرب میشه بره ولی به یه مشکلی بر میخورم و اونم اینه که نمیشه قدر مطلق خورشید رو گذاشت اون ور رابطه چون در اون صورت قدر مطلق کهکشان یه چیز تو اردر قدر مطلق خورشید در میاد :|
بعد این (http://up.avastarco.com/images/kxlmyzvq3gdno67ou2n.jpg)فرمول رو هم میشه اثبات کنین ؟

سلام!
اول بگم که من خودمم جوابش رو نمیدونم. اما شما یه اشتباهی کردی. دقت کنید که توی رابطه ی تولی فیشر ما سرعت چرخش کهکشان به دور خودش (یا بطور دقیقتر ماکسیمم این سرعت) رو داریم، نه سرعت چرخش یه ستاره. پس کلا باید توی تناسب سرعت چرخشی کهکشان و قدر مطلق کهکشان رو بذاریم، نه یه ستاره. اما نمیدونم این اطلاعات رو از کجا باید بیاریم!:grin:

m.Sadat
04-17-2014, 12:20 PM
خب، من فکر میکنم که جواب مستقل از عرض جغرافیایی است و در نیمکره شمالی در روز انقلاب تابستانی و در نیمکره جنوبی در روز انقلاب زمستانی این اتفاق می افتد.



سلام!
اول بگم که من خودمم جوابش رو نمیدونم. اما شما یه اشتباهی کردی. دقت کنید که توی رابطه ی تولی فیشر ما سرعت چرخش کهکشان به دور خودش (یا بطور دقیقتر ماکسیمم این سرعت) رو داریم، نه سرعت چرخش یه ستاره. پس کلا باید توی تناسب سرعت چرخشی کهکشان و قدر مطلق کهکشان رو بذاریم، نه یه ستاره. اما نمیدونم این اطلاعات رو از کجا باید بیاریم!:grin:

این صورت کامل سواله که توی سوال قید کرده که در روز اول تیر این اتفاق نمیفته

http://up.avastarco.com/images/on38zweq8zmmsnlpx9x1.png (http://up.avastarco.com/)

چیزی که واضح نیست اینه که باید اثر تعدیل زمان رو در نظر بگیریم یانه (نظر من اینه که باید در نظر گرفته شه )

univers
04-17-2014, 02:58 PM
راه حل این سوال آقای صدر الدینی در اینجا (http://forum.avastarco.com/forum/showthread.php?97-%D8%B1%D9%81%D8%B9-%D8%A7%D8%B4%D9%83%D8%A7%D9%84-%D8%A7%D9%84%D9%85%D9%BE%DB%8C%D8%A7%D8%AF-%D9%86%D8%AC%D9%88%D9%85/page36) موجود است.

univers
04-17-2014, 03:03 PM
این صورت کامل سواله که توی سوال قید کرده که در روز اول تیر این اتفاق نمیفته

http://up.avastarco.com/images/on38zweq8zmmsnlpx9x1.png (http://up.avastarco.com/)

چیزی که واضح نیست اینه که باید اثر تعدیل زمان رو در نظر بگیریم یانه (نظر من اینه که باید در نظر گرفته شه )
منم با نظر mohammad 1375 موافقم.اگر بدون هیچ فرضی جلو بریم مستقل از عرض جغرافیایی میشه که به نظرم غلطه.
اگه شفق رو درنظر بگیریم و حل کنیم به یک رابطه ساده و خوب میرسیم ولی برای تهران حل نمیشه.(سینوس فی بزرگتر از 1میشه)
کلا نمی دونم سوال چی فرض کرده.اساتید شما نظری ندارید؟

ali zeynali
04-18-2014, 01:15 AM
منم با نظر mohammad 1375 موافقم.اگر بدون هیچ فرضی جلو بریم مستقل از عرض جغرافیایی میشه که به نظرم غلطه.
اگه شفق رو درنظر بگیریم و حل کنیم به یک رابطه ساده و خوب میرسیم ولی برای تهران حل نمیشه.(سینوس فی بزرگتر از 1میشه)
کلا نمی دونم سوال چی فرض کرده.اساتید شما نظری ندارید؟
منظورش از زودتر اين نيست كه زاويه ساعتي خورشيد كمتر باشه! اينه كه ساعت مچي ما عدد كمتري رو نشون بده.

fatemeh.zar
04-18-2014, 07:39 AM
بنظرم برای عرض های بیشتر از 66.5 اون روزی میشه که میل برابر میشه با 90 منهای فی. و برای سایر عرض ها همون انقلاب تابستانی.

ali zeynali
04-18-2014, 09:40 AM
بنظرم برای عرض های بیشتر از 66.5 اون روزی میشه که میل برابر میشه با 90 منهای فی. و برای سایر عرض ها همون انقلاب تابستانی.
خب ميل كه از ٢٣.٥ بيشتر نميشه.
بايد تعديل زمان رو هم تاثير بدين.

1997
04-19-2014, 10:14 PM
سلام
گفتم نزدیک مرحله دو هستش و همه در حال پرسیدن اشکالات خودشون هستن.
منم سوال داشتم.اگه میشه جواب بدید.فقط خداکنه سوال قبل نشه که هیچ کس جواب نداد.
http://up.avastarco.com/images/93szztahzv68bzt1xq2b.png
کسی درباره ی سوال من نظری نداره؟
یه راهنمایی کنید بتونم حلش کنم.همه ی سوالا جواب داده شده به جز سوال من.

1997
04-19-2014, 10:27 PM
ببخشید اگه میشه یه جوابی هم به سوال من بدید یا حداقل راهنمایی کنید.
http://up.avastarco.com/images/93szztahzv68bzt1xq2b.png

mjaasgari
04-24-2014, 02:19 PM
سلام دوستان
اگه ممکنه به این چند سوال هم جواب بدید ممنون
(همه سوالات بجز سوال 4 که حل کردم تو فیلیپس هم هست.....)
http://www.xum.ir/images/2014/04/24/Photo_2014-04-24_025839_PM.jpg (http://www.xum.ir/images/2014/04/24/Photo_2014-04-24_025839_PM.jpg)

http://www.xum.ir/images/2014/04/24/Photo_2014-04-24_025851_PM.jpg (http://www.xum.ir/images/2014/04/24/Photo_2014-04-24_025851_PM.jpg)

arashgmn
04-25-2014, 12:41 AM
سلام دوستان
اگه ممکنه به این چند سوال هم جواب بدید ممنون
(همه سوالات بجز سوال 4 که حل کردم تو فیلیپس هم هست.....)
http://www.xum.ir/images/2014/04/24/Photo_2014-04-24_025839_PM.jpg (http://www.xum.ir/images/2014/04/24/Photo_2014-04-24_025839_PM.jpg)

http://www.xum.ir/images/2014/04/24/Photo_2014-04-24_025851_PM.jpg (http://www.xum.ir/images/2014/04/24/Photo_2014-04-24_025851_PM.jpg)

برای 5، می تونید دمای قطب و خود کوه رو جایگذاری کنید. تتاشون به ترتیب برابره با متمم عرض جغرافیایی و صفر. از روی این دو معادله، a,b تعیین میشن و ...

برای 3، می تونید از قاعده ی زنجیره ای برای r دبل دات استفاده کنید...( r-duble dot = (r-dot)(d(r-dot)/dr )

سوال 7 تو اسمارت هست. به تفصیل...

mjaasgari
04-25-2014, 08:50 AM
برای 5، می تونید دمای قطب و خود کوه رو جایگذاری کنید. تتاشون به ترتیب برابره با متمم عرض جغرافیایی و صفر. از روی این دو معادله، a,b تعیین میشن و ...

برای 3، می تونید از قاعده ی زنجیره ای برای r دبل دات استفاده کنید...( r-duble dot = (r-dot)(d(r-dot)/dr )

سوال 7 تو اسمارت هست. به تفصیل...

ببخشيد سوال ٥ ما عرض جغرافيايي نداريم كه بتونيم a&bرو پيدا كنيم؟
آ و ب برحسب عرض جغرافيايي بدست مياد
بعدش هم چطور ميتونم ميانگين رو بدست بيارم نميشه كه تمام نقاط دماشونو بدست بيارم؟ميشه؟
سوال هفت كدوم فصل اسمارت هست ميشه بگيد كجاسا؟
ممنون

tina
04-25-2014, 05:13 PM
ببخشيد سوال ٥ ما عرض جغرافيايي نداريم كه بتونيم a&bرو پيدا كنيم؟
آ و ب برحسب عرض جغرافيايي بدست مياد
بعدش هم چطور ميتونم ميانگين رو بدست بيارم نميشه كه تمام نقاط دماشونو بدست بيارم؟ميشه؟
سوال هفت كدوم فصل اسمارت هست ميشه بگيد كجاسا؟
ممنون

سلام :)
فکر کنم سوال 5 یه چیزی کم داشت باید رابطه ی یه چیزی رو با یه چیزی میداد که نداده ... (عجب جمله ی چیزی ! )

arashgmn
04-26-2014, 09:05 PM
ببخشيد سوال ٥ ما عرض جغرافيايي نداريم كه بتونيم a&bرو پيدا كنيم؟
آ و ب برحسب عرض جغرافيايي بدست مياد
بعدش هم چطور ميتونم ميانگين رو بدست بيارم نميشه كه تمام نقاط دماشونو بدست بيارم؟ميشه؟
سوال هفت كدوم فصل اسمارت هست ميشه بگيد كجاسا؟
ممنون
خوب الان مگه دمای قطب یه سینوس فی نداره و دمای استوا یه کسینوس فی ؟ اگه a و b رو برحسب فی درآریم، اصولا باید بشه دمای استوا رو پیدا کرد. باید بزنید تو سر اون سینوس و کسینوسه... :دی

یه فصل هم هست توی اسمارت به اسم اختلاف منظر. همه این ها رو تو ضیح داده. البته الان فقط باید مسئله حل کنید. وقت درس خوندن نیست...

tina
04-27-2014, 10:50 AM
سلام
اگه رابطه ی دوره تناوب یه ستاره ی تپنده با جرم و شعاعش (فرمول 14-14 مادرن)رو بدونیم چه جوری باید رابطه ی دوره تناوب- تابندگیشو (فرمول 2-14 مادرن) در بیاریم ؟
من وقتی مینویسم به یه مشت ثابت میرسم در شعاع ستاره به توان 9 :|

m.Sadat
04-27-2014, 11:50 AM
سلام
اگه رابطه ی دوره تناوب یه ستاره ی تپنده با جرم و شعاعش (فرمول 14-14 مادرن)رو بدونیم چه جوری باید رابطه ی دوره تناوب- تابندگیشو (فرمول 2-14 مادرن) در بیاریم ؟
من وقتی مینویسم به یه مشت ثابت میرسم در شعاع ستاره به توان 9 :|
بسته به این که چه فرضی میکنید جوابا متفاوته مثلا یکی از فرض ها اینه که توی اون ناحیه ای که قیفاوسی ها هستن دما تقریبا ثابته پس درخشندگی با شعاع به توان دو متناسب میشه ازاین جا رابطه جرم و شعاع درمیاد اگه بزاریم توی رابطه دوره تناوب نسبت دروه تناوب با شعاع بدست میاد و اگه دو باره توی رابطه استفان بولتزمن بزاریم رابطه درخشندگی و دوره تناوب رو بدست میاریم
اون رابطه اصلی هم با رصد بدست اومده :thumbsup:

tina
07-10-2014, 05:27 PM
سلام
داشتم فصل 3 کتاب استلار رو میخوندم که به یه تابع توزیعی رسیدم که نمیدونم اسمش چیه و از کجا اومده ! (خودم حدس زدم فرمی - دیراک باشه که تو مادرن گفته ولی وقتی تو اینترنت دیدم چیزی نفهمیدم:62: ) معادله ی 3.31 صفحه ی 51
http://s5.picofile.com/file/8129499526/%D8%B9%D8%AC%D8%A8.jpg
میشه راجع بهش توضیح بدین ؟:دی

arashgmn
07-10-2014, 07:35 PM
سلام
داشتم فصل 3 کتاب استلار رو میخوندم که به یه تابع توزیعی رسیدم که نمیدونم اسمش چیه و از کجا اومده ! (خودم حدس زدم فرمی - دیراک باشه که تو مادرن گفته ولی وقتی تو اینترنت دیدم چیزی نفهمیدم:62: ) معادله ی 3.31 صفحه ی 51
http://s5.picofile.com/file/8129499526/%D8%B9%D8%AC%D8%A8.jpg
میشه راجع بهش توضیح بدین ؟:دی

این اومده گفته که بر پایه ی اصل عدم قطعیت، می دونیم که دلتا ایکس × دلتا تکانه(ی راستای ایکس) >= h

این موضوع برای مواد کوانتومی در هر سه بعد، برقراره. پس اگه بیایم سه معادله ی مربوط به سه بعد رو در هم ضرب کنیم، میشه این:

delta(x) delta(y) delta(z) delta(Px) delta(Py) delta(pz) >= h.h.h

ضرب دلتاهای مکان رو به صورت (delta(V نوشته و ضرب دلتاهای تکانه رو توی دستگاه کروی نوشته (ضرب دلتاهای تکانه، مثل دیفرانسیل حجم یه فضاییه که محورهاش تکانه تو راستاهای x و y و z هستن...)

حالا اومده گفته این حجم دلتا وی ، مثل یه اوربیتاله. پس اگه الکترون ها کوانتومی باشن، داخل هر اوربیتال، فقط 2 تا الکترون داریم. بعد گفته که ما برای حالت حدی معادلات رو می نویسیم. پس چگالی تعداد الکترون هایی که تکانه ای بین p و p+dp دارن و توی این اوربیتال میشه 2 تقسیم بر دلتا وی. که میشه همون عبارت بالا به شرطی که به جای علامت بزرگتر مساوی، مساوی گذاشته بشه.

tina
07-10-2014, 08:31 PM
آها ...
ولی مشکل اصلیم با دومین مساویه
مثلا چهار پی × تکانه به توان دو از کجا اومده ؟:slow:

starscream4002
07-10-2014, 09:40 PM
خوب قبول دارین که در دستگاه مختصات کروی، (سه بعدی) دیفرانسیل حجم برابر چهار پی ضربدر r به توان 2 ضربدر دیفرانسیل r هست؟ (که r می شه رادیکال، x به توان 2 به علاوه y به توان 2 به علاوه z به توان 2) خوب اینجا تکانه ها هم 3 تا محور مختصات از فضای 6 بعدی فاز رو تشکیل می دن. پس دیفرانسیل حجمشون می شه 4 پی ضربدر p به توان 2 ضربدر dp! که در اینجا، مثل دستگاه مختصات کروی، تکانه x به توان 2 به علاوه ی تکانه y به توان 2 به علاوه ی تکانه z به توان 2 می شه تکانه کل به توان 2! پس می شه اون رو مثل یه دستگاه کروی فرض کرد.

tina
07-10-2014, 10:09 PM
اَ :دی
یعنی چون ما تغییرات تکانه رو میخوایم و این تغییرات تو سه بعده شبیه یه حجم در نظرش میگیریم ؟ :دی
خیلی جالب بود ایده اش :دی

starscream4002
07-10-2014, 10:27 PM
اَ :دی
یعنی چون ما تغییرات تکانه رو میخوایم و این تغییرات تو سه بعده شبیه یه حجم در نظرش میگیریم ؟ :دی
خیلی جالب بود ایده اش :دی

دقیقا! درست مثل یه حجم با سه بعد تکانه ای مون رفتار می کنیم.

fatemeh.zar
07-17-2014, 11:11 AM
سلام!
خب تصمیم گرفتیم چندی از اشکالات خویش را اینجا بپرسیم!
سوال 2 فصل اول ماوز
http://up.avastarco.com/images/7g5u7ez3u9z9v01a09n.gif
این سیگنال به نویز چه ربطی به seeing داره؟؟ باید کل سیگنال رو تقسیم بر پیکسل های seeing کنیم یعنی؟؟
بعد نقش دهانه تلسکوپ چیه کلا؟
این یکی هم سوال یک فصل 2:
http://up.avastarco.com/images/wqr7w0xpyu08b61n0sr.jpg
چجوری باید شار بولومتریک رو از روی شار تک طول موجی حساب کرد؟

mahdad_haghighi
07-17-2014, 02:02 PM
سلام!
خب تصمیم گرفتیم چندی از اشکالات خویش را اینجا بپرسیم!
سوال 2 فصل اول ماوز
این سیگنال به نویز چه ربطی به seeing داره؟؟ باید کل سیگنال رو تقسیم بر پیکسل های seeing کنیم یعنی؟؟
بعد نقش دهانه تلسکوپ چیه کلا؟

سلام
S/N ربط مستقیمی به Seeing نداره! اثرات جو یه شعاعی برای این نور این ستاره ایجاد کرده که سوال خواسته S/N تو اون شعاع محاسبه بشه.
سیگنال به نویز یه تعریف داره که از توزیع پواسون به دست میاد (اگه دوست داشتین بدونین دقیقا چیه و از کجا میاد :http://www.ucolick.org/~bolte/AY257/s_n.pdf و http://en.wikipedia.org/wiki/Signal-to-noise_ratio)
(Signal-to-noise ratio=N/((N+n)^1/2
که تو این سوال چون n رو واسه ی یه پیکسل داده ، شعاع داده شده تو سوال هم مورد استفاده قرار می گیره.
دو تا عامل تو تعیین شعاع اون دایره تاثیر دارن (یکی پارامتر Seeing و یکی قدرت تفکیک تلسکوپ)، احتمالا قطر دهانه تلسکوپ برای محاسبه عامل دوم نوشته شده! (ویرایش: این موضوع در حالت کلی هست ولی فک کنم واسه این سوال اشتباه گفتم این رو :دی)
دوستان لطفا هرجا رو اشتباه گفتم اصلاح کنن :)

mahdad_haghighi
07-17-2014, 02:31 PM
چجوری باید شار بولومتریک رو از روی شار تک طول موجی حساب کرد؟

نیازی به محاسبه شار بولومتریک نیست! تمام کارهایی رو که در قسمت a سوال انجام دادید تکرار کنید فقط با این تفاوت که در آخرین مرحله شار تک طول موجی رو برابر معادله 2.5 کتاب بزارید. تو عبارتی که به دست میاد فقط T مجهوله که خواسته سواله، اگر واضح نبود توضیحم بگین کامل بنویسم راه حل رو :)

fatemeh.zar
07-18-2014, 11:34 AM
سلام
S/N ربط مستقیمی به Seeing نداره! اثرات جو یه شعاعی برای این نور این ستاره ایجاد کرده که سوال خواسته S/N تو اون شعاع محاسبه بشه.
سیگنال به نویز یه تعریف داره که از توزیع پواسون به دست میاد (اگه دوست داشتین بدونین دقیقا چیه و از کجا میاد :http://www.ucolick.org/~bolte/AY257/s_n.pdf و http://en.wikipedia.org/wiki/Signal-to-noise_ratio)
(Signal-to-noise ratio=N/((N+n)^1/2
که تو این سوال چون n رو واسه ی یه پیکسل داده ، شعاع داده شده تو سوال هم مورد استفاده قرار می گیره.
دو تا عامل تو تعیین شعاع اون دایره تاثیر دارن (یکی پارامتر Seeing و یکی قدرت تفکیک تلسکوپ)، احتمالا قطر دهانه تلسکوپ برای محاسبه عامل دوم نوشته شده! (ویرایش: این موضوع در حالت کلی هست ولی فک کنم واسه این سوال اشتباه گفتم این رو :دی)
دوستان لطفا هرجا رو اشتباه گفتم اصلاح کنن :)

ولی من حدس میزنم دهانه رو برای این داده که هرچی دهانه بزرگتر باشه، قدرت جمع آوری نور بیشتره پس سیگنال بیشتره. یعنی توی حالت دوم دیگه سیگنال n_star نیست. نسبت سیگنال هاش هم بشه نسبت مربع دهانه ها.
البته فقط حدسه! :)

Pale_Rider
07-19-2014, 10:37 PM
در اصل فک کنم عدم قطعیت هایزنبرگ یه ژوب به حساب می آد چون تا جایی که می دونم عدم قطعیت خود ثابت پلانک نیست و ثابت پلانک کاهیده هستش ، اگه منفی و مثبت فضای فاز هم در نظر بگیریم یه عددی (حدود 4 ) اضافی می آد که با وجود دقتی که داریم ( اسپین هم جلوتر وارد می شه ) اصلا نمی تونیم از 4 چشم پوشی کنیم ، اثبات درستش هم فک کنم با الکترومغناطیس و معادله موج هستش ، که اول ضریب k ( تو معادله ی موج : exp(i(kx-wt))P0 ) رو با توجه به یک حجم 3 بعدی مکان خاص به دست می آریم و از خاصیت موجی ذره ( اصل دوبروی ) طول موج کوانتومی ذره رو حساب می کنیم و به dk^3 می رسیم ، بعد اون رو می نویسیم 4pi*k^2*dk و بعد هم که با الکترومغناطیس k رو بدست می آریم ، در اصل ایده نیست ، اسمش فضای فاز هستش و واقعا هم یک حجم 6 بعدی داریم که 3 بعد تکانه و 3 بعد مکان (مختصات تعمیم یافته (با کمک narcisus flower عزیز ! )) داره اگه 2 تا ذره داخل این حجم باشن نمی تونیم از همدیگه تفکیکشون کنیم ( در اصل یه جورایی خطا از ما نیست مشکل از طبیعته ! )
حالا برای حساب کردن تعداد ذرات می بینیم که چند تا از این فضاها داریم ! بعد تعداد فضاها رو ضرب می کنیم در تعداد ذرات در هر فضا که بسته به ذره از توزیع فرمی-دیراک یا بوز-انیشتین تبعیت می کنه و خیلی ساده تعداد بدست می آد !
در ضمن اگه کسی فک می کنه عدم قطعیت ژوب نیست لطفا منو قانع کنه !

mahdad_haghighi
07-19-2014, 11:10 PM
در اصل فک کنم عدم قطعیت هایزنبرگ یه ژوب به حساب می آد چون تا جایی که می دونم عدم قطعیت خود ثابت پلانک نیست و ثابت پلانک کاهیده هستش ، اگه منفی و مثبت فضای فاز هم در نظر بگیریم یه عددی (حدود 4 ) اضافی می آد که با وجود دقتی که داریم ( اسپین هم جلوتر وارد می شه ) اصلا نمی تونیم از 4 چشم پوشی کنیم ، اثبات درستش هم فک کنم با الکترومغناطیس و معادله موج هستش ، که اول ضریب k ( تو معادله ی موج : exp(i(kx-wt))P0 ) رو با توجه به یک حجم 3 بعدی مکان خاص به دست می آریم و از خاصیت موجی ذره ( اصل دوبروی ) طول موج کوانتومی ذره رو حساب می کنیم و به dk^3 می رسیم ، بعد اون رو می نویسیم 4pi*k^2*dk و بعد هم که با الکترومغناطیس k رو بدست می آریم ، در اصل ایده نیست ، اسمش فضای فاز هستش و واقعا هم یک حجم 6 بعدی داریم که 3 بعد تکانه و 3 بعد مکان (مختصات تعمیم یافته (با کمک narcisus flower عزیز ! )) داره اگه 2 تا ذره داخل این حجم باشن نمی تونیم از همدیگه تفکیکشون کنیم ( در اصل یه جورایی خطا از ما نیست مشکل از طبیعته ! )
حالا برای حساب کردن تعداد ذرات می بینیم که چند تا از این فضاها داریم ! بعد تعداد فضاها رو ضرب می کنیم در تعداد ذرات در هر فضا که بسته به ذره از توزیع فرمی-دیراک یا بوز-انیشتین تبعیت می کنه و خیلی ساده تعداد بدست می آد !
در ضمن اگه کسی فک می کنه عدم قطعیت ژوب نیست لطفا منو قانع کنه !
فک می کنم تا حد زیادی درست می گی و اصل عدم قطعیت هایزنبرگ جواب نمیده، چون اگر تکانه فوتون رو بزاریم باید به شار گاز فوتونی رسید و همون طوری که می دونیم توی تابع پلانک، h داریم نه h بار!
سر کلاس دکتر هم سر استدلال های فیلیپس و هارویت خیلی بحث کردیم ولی خب قانع نشدیم پس اگه قانع شدی منم قانع کن لطفا :دی

tina
07-25-2014, 12:12 PM
سلام
من تو فصل 14 مادرن ،اون جایی که میخواد دوره تناوب نوسان ستاره رو حساب کنه (http://up.avastarco.com/images/dhuj5sycxliupmu52rc9.jpg)، ایراد دارم ...
انتگرالشو که میخوام حل کنم به انتگرال سکانت میرسم (که البته خودم هم بلد نبودم حل کنم ولی سه تا راه مختلف براش پیدا کردم ... این (http://up.avastarco.com/images/azyousr23ulpc8emmiav.jpg) و این (http://up.avastarco.com/images/t27dl22p3iae727tc5zx.jpg)و این (http://up.avastarco.com/images/azyousr23ulpc8emmiav.jpg))که وقتی حدشو از پی دوم تا صفر میذارم مخرج کسر یه جا صفر میشه و حساب نمیشه میشه راهنمایی کنین چجوری حل میشه ؟
[البته میدونم سوالی که پرسیدم بیشتر ریاضیه تا نجوم ... :26:]

solh
07-25-2014, 01:02 PM
سلام
من تو فصل 14 مادرن ،اون جایی که میخواد دوره تناوب نوسان ستاره رو حساب کنه (http://up.avastarco.com/images/dhuj5sycxliupmu52rc9.jpg)، ایراد دارم ...
انتگرالشو که میخوام حل کنم به انتگرال سکانت میرسم (که البته خودم هم بلد نبودم حل کنم ولی سه تا راه مختلف براش پیدا کردم ... این (http://up.avastarco.com/images/azyousr23ulpc8emmiav.jpg) و این (http://up.avastarco.com/images/t27dl22p3iae727tc5zx.jpg)و این (http://up.avastarco.com/images/azyousr23ulpc8emmiav.jpg))که وقتی حدشو از پی دوم تا صفر میذارم مخرج کسر یه جا صفر میشه و حساب نمیشه میشه راهنمایی کنین چجوری حل میشه ؟
[البته میدونم سوالی که پرسیدم بیشتر ریاضیه تا نجوم ... :26:]
http://up.avastarco.com/images/71m7sx0ik6a7v16m0cp.gif
منفی رو یکی دوجا نذاشتم که با هم خط خوردن !

tina
08-11-2014, 01:37 PM
سلام :15:
توی این (http://up.avastarco.com/images/ry2wpppeyd7h7m6de2xz.jpg) سواله من قسمت آخرش (مثه همیشه تو انتگرالش) ایراد دارم
این (http://up.avastarco.com/images/yreobcw623d7b7g9pfab.jpg) راهنمایی خودشه اینم (http://up.avastarco.com/images/qjbvxyjovm1byw5qb41q.jpg) راه منه ... تهش یه چیزایی اضافه میارم :| کجاشو دارم اشتباه میکنم ؟ :39:

fatemeh.zar
08-11-2014, 05:41 PM
سلام :15:
توی این (http://up.avastarco.com/images/ry2wpppeyd7h7m6de2xz.jpg) سواله من قسمت آخرش (مثه همیشه تو انتگرالش) ایراد دارم
این (http://up.avastarco.com/images/yreobcw623d7b7g9pfab.jpg) راهنمایی خودشه اینم (http://up.avastarco.com/images/qjbvxyjovm1byw5qb41q.jpg) راه منه ... تهش یه چیزایی اضافه میارم :| کجاشو دارم اشتباه میکنم ؟ :39:

سلام:)
من حال نداشتم راه حلتو بخونم ولی این راه منه:
http://up.avastarco.com/images/ai0cf5rwlg7kd777zrx.jpg (http://up.avastarco.com/)http://up.avastarco.com/images/n83wyeg633h4jey92hi2.jpg (http://up.avastarco.com/)

tina
08-12-2014, 10:28 AM
اممم ...
خوب الان فهمیدم مشکلم تو تصویر کردنه حس میکنم باید یه ضریبی اضافه باشه ...
الان میرم فک میکنم ببینم چرا نباید باشه ...
مرسی ;)

tina
09-04-2014, 09:53 AM
سلام
من تو سوال 8 فصل یک فیلیپس مشکل دارم
قسمت دوم سوال گفته ثابت کنید نمودار به صورت یک خط راست است . (که خوب من با این (http://up.avastarco.com/images/gaewbkr81enjpm57hs0u.jpg) راهم به خط راست نمیرسم .)
بعد قسمت سوم میگه شیب خط رو برای سنگ ها با سن های مختلف به دست بیارین .
مگه اگه خط راست باشه شیب همه نباید یکی باشه ؟ :|
[الان دیدم ته کتاب راهنمایی داره ؛ یا تعریف من از خط راست غلطه یا فیلیپس :/ ]

fatemeh.zar
09-05-2014, 12:40 PM
سلام
من تو سوال 8 فصل یک فیلیپس مشکل دارم
قسمت دوم سوال گفته ثابت کنید نمودار به صورت یک خط راست است . (که خوب من با این (http://up.avastarco.com/images/gaewbkr81enjpm57hs0u.jpg) راهم به خط راست نمیرسم .)
بعد قسمت سوم میگه شیب خط رو برای سنگ ها با سن های مختلف به دست بیارین .
مگه اگه خط راست باشه شیب همه نباید یکی باشه ؟ :|
[الان دیدم ته کتاب راهنمایی داره ؛ یا تعریف من از خط راست غلطه یا فیلیپس :/ ]

سلام!
تو خط اول راهت n05 رو بر حسب nt5 جایگذاری کن میرسی به فرمولی که تو صورت سوال داده. بعد از رو اون nt7 بر حسب nt6 یه تابع درجه یک میشه که ضریب n6t خودش تابعی از t هست( (f(t). برای سنگهایی که در زمان یکسان تولید شدند t یکیه بنابراین (f (t هم ثابته و نمودار خطی میشه. دقت کن که (f(t برای سنگهایی که یه میلیارد سال پیش بوجود اومدن ثابته، برای اونایی که سه میلیارد سال پیش بوجود اومدن هم ثابته؛ اما این دو تا ثابت با هم برابر نیستن!!

mh1375
09-05-2014, 06:01 PM
سلام
سوتی شما تو متن سواله...وقتی نوشته تو پرانتز 3.13 که چگالی (R^2+Z^2) زیر رادیکال منظورش ضرب این پرانتز در چگالی نیست...یعنی چگالی تابعی از r است همین و بس!
بقیه اش هم که خانم fatemeh.zar نوشتن...
یاعلی

tina
09-05-2014, 08:41 PM
سلام!
تو خط اول راهت n05 رو بر حسب nt5 جایگذاری کن میرسی به فرمولی که تو صورت سوال داده. بعد از رو اون nt7 بر حسب nt6 یه تابع درجه یک میشه که ضریب n6t خودش تابعی از t هست( (f(t). برای سنگهایی که در زمان یکسان تولید شدند t یکیه بنابراین (f (t هم ثابته و نمودار خطی میشه. دقت کن که (f(t برای سنگهایی که یه میلیارد سال پیش بوجود اومدن ثابته، برای اونایی که سه میلیارد سال پیش بوجود اومدن هم ثابته؛ اما این دو تا ثابت با هم برابر نیستن!!
آهــــــــــــــا ! منظورش اینه برا یه سری سنگ هم سن کشیدن !
من فکر کردم برای یه سنگ توی چند میلیار سال کشیدن :| (واقعا با خودم فکر نکردم که هیچ دانشمندی از چند میلیار سال پیش نَشِسته برای یه سنگ نمودار بکشه ؟ :دی )

سلام
سوتی شما تو متن سواله...وقتی نوشته تو پرانتز 3.13 که چگالی (R^2+Z^2) زیر رادیکال منظورش ضرب این پرانتز در چگالی نیست...یعنی چگالی تابعی از r است همین و بس!
بقیه اش هم که خانم fatemeh.zar نوشتن...
یاعلی
آره
اون موقع فهمیدم ولی این قدر سوتی گندی بود ترجیح دادم به روی خودم نیارم بندازم گردن تصویر کردن ...
متاسفانه بعد از مدتی هم ویرایش خراب میشه وگرنه میرفتم نیست و نابود میکردم اون فایل رو :|

tina
09-07-2014, 07:43 PM
سلام .
تو این (http://up.avastarco.com/images/se8ewge81t30u76blokg.jpg)سواله چجوری باید شعاع خوشه رو در بیارم ؟
اصلا از جرم ویریالی باید برم که به شعاع نیازی باشه ؟ :/
میخواستم از چگالی پلامر در چهار پی آر دو دی (آر) از صفر تا آر انتگرال بگیرم ولی این جوری در نیومد :( این اِمی که تو چگالی پلامره جرم کُله ؟ :/

fatemeh.zar
09-07-2014, 09:17 PM
سلام .
تو این (http://up.avastarco.com/images/se8ewge81t30u76blokg.jpg)سواله چجوری باید شعاع خوشه رو در بیارم ؟
اصلا از جرم ویریالی باید برم که به شعاع نیازی باشه ؟ :/
میخواستم از چگالی پلامر در چهار پی آر دو دی (آر) از صفر تا آر انتگرال بگیرم ولی این جوری در نیومد :( این اِمی که تو چگالی پلامره جرم کُله ؟ :/
و علیک السلام یا تینا!
نیازی به شعاع خوشه نیست!
توی سوال 3.2 بدست میاریم که m همون جرم کل هست.
توی سوال 3.12 هم انرژی پتانسیل برای چگالی پلامر بدست میاد که توی رابطه اش چیزی به نام شعاع نداره!
حالا کافیه ویریال بزنی به رگ!
کلا سوال 3.2 همه چی چگالی پلامر رو گفته حساب کنید. اگه حل نکردی حل کن بدرد میخوره:)

tina
09-08-2014, 09:03 AM
و علیک السلام یا تینا!
نیازی به شعاع خوشه نیست!
توی سوال 3.2 بدست میاریم که m همون جرم کل هست.
توی سوال 3.12 هم انرژی پتانسیل برای چگالی پلامر بدست میاد که توی رابطه اش چیزی به نام شعاع نداره!
حالا کافیه ویریال بزنی به رگ!
کلا سوال 3.2 همه چی چگالی پلامر رو گفته حساب کنید. اگه حل نکردی حل کن بدرد میخوره:)

:دی
آها ، خوب 3.12 رو حل نکردم الان میرم حلش میکنم . الان به یه نتیجه ای رسیدم و اون اینه که نمیتونیم از ام ویریالی ای که تو مادرن برا حساب کردن جرم سیاهچاله داده استفاده کنیم :دی چون این پتانسیلش فرق داره اون که با فرض پلامر حل نکرده بود :دی (خسته نباشم که تازه به این نتیجه رسیدم ولی تازه رسیدم دیگه چه کنم ؟ ((: ) . 3.2 رو مثلا حل کردم ولی باز حلش میکنم :دی برم بزنمشون در رگ جان . مرسی ؛؛)

fatemeh.zar
09-10-2014, 10:44 AM
:دی
آها ، خوب 3.12 رو حل نکردم الان میرم حلش میکنم . الان به یه نتیجه ای رسیدم و اون اینه که نمیتونیم از ام ویریالی ای که تو مادرن برا حساب کردن جرم سیاهچاله داده استفاده کنیم :دی چون این پتانسیلش فرق داره اون که با فرض پلامر حل نکرده بود :دی (خسته نباشم که تازه به این نتیجه رسیدم ولی تازه رسیدم دیگه چه کنم ؟ ((: ) . 3.2 رو مثلا حل کردم ولی باز حلش میکنم :دی برم بزنمشون در رگ جان . مرسی ؛؛)

البته من مشکلی نمی بینم که از ویریال به صورت k+2u=0 استفاده کنیم. چون تو اثباتش شکل نیرو و یا پتانسیل دخالتی نداشت.

mh1375
09-10-2014, 11:12 AM
برای استفاده از ویریال همون طور که گفتن موردی نداره فقط برای پتانسیل گرانشی(u) و جنبشی(k) لازمه برای هر پتانسیل منحصر به فرد از روابط زیر استفاده کنین:
3.33
و اینکه سرعت^2 تقسیم بر فاصله می شود منفی مشتق پتانسیل نسبت به فاصله

راستی یه سوال سر کاری ام میگم روش فک کنین چرا اگه از 3.33 بریم برای پتانسیل نیوتنی بریم سه پنجم معروف در نمیاد؟؟؟
یه چیز دیگه ام اینکه حواستون باشه که ویریال همیشه مساوی صفر نیست (اون طرف مشتق دوم زمانی ممان اینرسی(i) هست)ولی ما میتونیم تو هر لحظه سیستم رو تقریبا ایزوله فرض کنیم...
موفق باشید
یاعلی

mohammadmehdi
10-05-2014, 06:40 PM
سلام حل المساءل مجموعه سوالات وجود ندارن؟؟
من کلی اشکال توشون دارم!!!

fatemeh.zar
10-06-2014, 04:30 PM
سلام حل المساءل مجموعه سوالات وجود ندارن؟؟
من کلی اشکال توشون دارم!!!

سلام!
تا جایی که من میدونم نه!
میتونید اینجا اشکالاتون رو بپرسید تا من و بقیه دوستان کمکتون کنیم.

tina
10-06-2014, 05:51 PM
سلام (:
فصل دو فیلیپس فرمول 2-7 : http://up.avastarco.com/images/us6u31e3sdgptpxnmx.jpg

در مورد آنتروپی توی مادرن یه توضیحی داده که من خیلی خوب متوجه نشدم ( یعنی اصلا متوجه نشدم ) چی هست ، کاربردش چیه و کجا به درد میخوره ؟
میو چیه ؟

arashgmn
10-06-2014, 09:21 PM
سلام (:
فصل دو فیلیپس فرمول 2-7 : http://up.avastarco.com/images/us6u31e3sdgptpxnmx.jpg

در مورد آنتروپی توی مادرن یه توضیحی داده که من خیلی خوب متوجه نشدم ( یعنی اصلا متوجه نشدم ) چی هست ، کاربردش چیه و کجا به درد میخوره ؟
میو چیه ؟

سلام.

آنتروپی، یه مفهوم (نسبتا سخت) ترمودینامیکیه که نشون دهنده‌ی میران بی‌نظمی یه سامانه‌اس. حقیقتش چیز زیادی نمی‌دونم که درباره‌اش بگم... . همونی که تو مادرن راجع بهش گفته، تو اغلب کتاب‌های فیزیک پایه‌ هم تا همون حد (یکم بیشتر) می‌گن.

میو رو هم نمی‌دونم. خودش قبل‌تر، تعریفش نکرده؟ N چیه؟ تعداد ذرات؟ یا چگالی عددیشون؟ شاید با تحلیل ابعادی بشه چیزی فهمید...! o_O

fatemeh.zar
10-06-2014, 09:28 PM
سلام
n فکر کنم تعداد ذرات هست و میو هم پتانسیل شیمیاییه که نمیدونم چیه!

mohammadmehdi
10-06-2014, 10:38 PM
ای فرمولو کجای مادرن بود؟؟

tina
10-07-2014, 05:32 PM
سلام.

آنتروپی، یه مفهوم (نسبتا سخت) ترمودینامیکیه که نشون دهنده‌ی میران بی‌نظمی یه سامانه‌اس. حقیقتش چیز زیادی نمی‌دونم که درباره‌اش بگم... . همونی که تو مادرن راجع بهش گفته، تو اغلب کتاب‌های فیزیک پایه‌ هم تا همون حد (یکم بیشتر) می‌گن.

میو رو هم نمی‌دونم. خودش قبل‌تر، تعریفش نکرده؟ N چیه؟ تعداد ذرات؟ یا چگالی عددیشون؟ شاید با تحلیل ابعادی بشه چیزی فهمید...! o_O

دی ان تغییر تعداد ذراته گویا ؛ میو هم که فاطمه گفتن ! پتانسیل شیمیایی ئه :/
( فک کنم فعلا بهتره ول کنم این فرمولو ؛ یکم که خیالم راحت شد برگردم روش ... )

ای فرمولو کجای مادرن بود؟؟

تو مادرن نیست .
نوشتم فصل دو فیلیپس که ! آنتروپی تو مادرن ، فصل 10 ، هست .

m.Sadat
10-07-2014, 07:36 PM
این قانون که چیز خاصی نداره همون قانون اول ترمودینامیکه با توجه به تعریف انتروپی Tds همون dQ ه و dN هم تغییر در نعداد ذراته سیستمه و میو هم پتانسیل شیمیایی ه که به ویژگی های سیستم بستگی داره و میشه برحسب دما و ... حسابش کرد مثلا برای فوتون صفره این میو همونه که توی تابع توزیع فرمی دیراک و بوز انیشتین هست
برای مطالعه بیشتر راجع به انتروپی و اینا هم میتونید به کتاب ترمودینامیک زیمانسکی مراجعه کنید

mohammadmehdi
10-08-2014, 05:04 PM
این سوال ٣ آزمون یک مجموعه سوالات آقای احسان عابدیه
چجوری حل میشه؟http://up.avastarco.com/images/tl6ai0kpbe1rgpcy14sq.jpg

mohammadmehdi
10-08-2014, 07:14 PM
ببخشید عکس قبلی مشکل داشت
این لینکه درسته
http://8pic.ir/images/81ulr3qzzyrv6y4ojn1s.jpg

fatemeh.zar
10-08-2014, 08:38 PM
سلام
یه دستگاه مختصات به مرکز مرکز بیضی میذاریم. مختصات زمین رو توش بدست میاریم. شیب خطی که از زمین و نقطه ای دلخواه روی بیضی می گذره رو پارامتری حساب می کنیم و با شیب مماس بر بیضی در اون نقطه مساوی قرار میدیم. دو تا جواب میده که یکیشون کشیدگی غربیه یکیشون شرقی. دیگه محاسبه بیهنجاری اش هم که با داشتن مختصات راحته.
برای حساب کردن خود کشیدگی ها هم کافیه به مثلث زمین-عطارد-خورشید توجه کنیم!!
اگر لازم بود بگید راهم رو کامل بنویسم. البته شاید راه های آسون تری هم داشته باشه این سوال!

mohammadmehdi
10-08-2014, 10:04 PM
سلام
یه دستگاه مختصات به مرکز مرکز بیضی میذاریم. مختصات زمین رو توش بدست میاریم. شیب خطی که از زمین و نقطه ای دلخواه روی بیضی می گذره رو پارامتری حساب می کنیم و با شیب مماس بر بیضی در اون نقطه مساوی قرار میدیم. دو تا جواب میده که یکیشون کشیدگی غربیه یکیشون شرقی. دیگه محاسبه بیهنجاری اش هم که با داشتن مختصات راحته.
برای حساب کردن خود کشیدگی ها هم کافیه به مثلث زمین-عطارد-خورشید توجه کنیم!!
اگر لازم بود بگید راهم رو کامل بنویسم. البته شاید راه های آسون تری هم داشته باشه این سوال!
ممنون!!!
دلیلش چیه که وقتی این دو تا رو مساوی میذاریم کشیدگی غربی و شرقی به دست میان؟

fatemeh.zar
10-10-2014, 06:45 PM
چون در موقع بیشترین کشیدگی خط واصل عطارد-زمین بر مدار عطارد مماسه. فکر کنم توی این شکل واضح باشه چرا مماس:

http://up.avastarco.com/images/kppztwyyje7yba9mlra.jpg (http://up.avastarco.com/)

mohammadmehdi
10-10-2014, 08:07 PM
چون در موقع بیشترین کشیدگی خط واصل عطارد-زمین بر مدار عطارد مماسه. فکر کنم توی این شکل واضح باشه چرا مماس:

http://up.avastarco.com/images/kppztwyyje7yba9mlra.jpg (http://up.avastarco.com/)

خب این درسته ولی شما گفتین شیب خط رو در هر نقطه ی دلخواه به دست بیاریم یعنی که مثلا میتونیم خط واصل زمین خورشید رو بگیریم؟؟
اگه نمیشه پس چطوری شیب مماس بر بیضی رو در کشیدگی حساب کنیم وقتی تتا رو نداریم؟

fatemeh.zar
10-20-2014, 09:33 PM
خب این درسته ولی شما گفتین شیب خط رو در هر نقطه ی دلخواه به دست بیاریم یعنی که مثلا میتونیم خط واصل زمین خورشید رو بگیریم؟؟
اگه نمیشه پس چطوری شیب مماس بر بیضی رو در کشیدگی حساب کنیم وقتی تتا رو نداریم؟

منظورم این بود که پارامتری بدست بیاریم. یعنی بر حسب x و y اون نقطه. هم شیب مماس هم شیب خط واصل پارامتری بدست میان، مساوی قرارشون میدیم و معادله حل میکنیم و الخ.

tina
10-23-2014, 12:25 AM
سلام
من دو تا سوال دارم
اولیش سوال 5 فصل 4 کتاب رایدنه .
من به جای اینکه 0 و 1/3 در بیارم 1- و 2/3- در میارم .
اینم راهمه (http://up.avastarco.com/images/lpxibak87625jhu9bu2x.jpg)
دومیش هم یه مفهومه .
من مفهوم شعاع عالم رو درک نمیکنم . مگه نمیگن جهان همگن و همسانگرده پس هیچ جا نسبت به هیچ جای دیگه برتری ای نداره ؟ خوب اگه این جوریه یه نقطه رو که مرکز مثلا کره میگیریم تا شعاع رو نسبت بهش بسنجیم یعنی بین اون نقطه و نقطه های دیگه فرق گذاشتیم دیگه ؟ :/

starscream4002
10-23-2014, 04:44 PM
سلام
من دو تا سوال دارم
اولیش سوال 5 فصل 4 کتاب رایدنه .
من به جای اینکه 0 و 1/3 در بیارم 1- و 2/3- در میارم .
اینم راهمه (http://up.avastarco.com/images/lpxibak87625jhu9bu2x.jpg)
دومیش هم یه مفهومه .
من مفهوم شعاع عالم رو درک نمیکنم . مگه نمیگن جهان همگن و همسانگرده پس هیچ جا نسبت به هیچ جای دیگه برتری ای نداره ؟ خوب اگه این جوریه یه نقطه رو که مرکز مثلا کره میگیریم تا شعاع رو نسبت بهش بسنجیم یعنی بین اون نقطه و نقطه های دیگه فرق گذاشتیم دیگه ؟ :/

سلام،

(از طرف من و محمد نبی زاده که جوابتون رو بررسی کردیم.)
راجع به سوال 5 فصل 4 رایدن، آیا وقتی a تغییر می کند، n ثابت می ماند؟ n خودش متناسب با a به توان منفی 3 هست و آن رو تاثیر بدهید، جواب درست می شود.
اصولا مفهومی به اسم شعاع عالم نداریم. هر چیزی که استفاده می کنیم صرفا مقیاسی از ابعاد عالم هست. ولی رایدن تقریبی از درجه مقیاس/ابعاد عالم را به شکل c/H0 معرفی کرده.
برای هر نقطه ای ما می توانیم یک افق رویداد و افق ذره تعریف کنیم (در صورت وجود. چرا؟) که به همین دلیل هیچ نقطه ای نسبت به نقاط دیگر برتری ندارد. (افق رویداد و ذره است که تعریف می شود. نه شعاع عالم.)

موفق باشید

tina
10-25-2014, 08:33 PM
سلام،

(از طرف من و محمد نبی زاده که جوابتون رو بررسی کردیم.)
راجع به سوال 5 فصل 4 رایدن، آیا وقتی a تغییر می کند، n ثابت می ماند؟ n خودش متناسب با a به توان منفی 3 هست و آن رو تاثیر بدهید، جواب درست می شود.
اصولا مفهومی به اسم شعاع عالم نداریم. هر چیزی که استفاده می کنیم صرفا مقیاسی از ابعاد عالم هست. ولی رایدن تقریبی از درجه مقیاس/ابعاد عالم را به شکل c/H0 معرفی کرده.
برای هر نقطه ای ما می توانیم یک افق رویداد و افق ذره تعریف کنیم (در صورت وجود. چرا؟) که به همین دلیل هیچ نقطه ای نسبت به نقاط دیگر برتری ندارد. (افق رویداد و ذره است که تعریف می شود. نه شعاع عالم.)

موفق باشید
مرسی :grin:فک کنم حالا (http://up.avastarco.com/images/4pfq5f6g2o7skcwqe74m.jpg)درست شد .
در مورد چرایی که پرسیدین الان جوابی بلد نیستم ولی یه هفته دیگه ایشالله اگه فهمیدم جوابشو میگم :دی ( لطفا خودتون نگین :دی )
الان دیدم یه جایی تو راهم به جای ام دو سی چهار گذاشتم ام دو سی دو که البته بقیه اش رو درست نوشتم دیگه حال ندارم عکسشو ویرایش کنم :دی

tina
10-26-2014, 10:31 AM
خوب من الان فهمیدم چند تا چیز دیگه رو هم خوب متوجه نشدم :
من فکر میکردم فقط برای جهان تک جزئی انحنا دار میگیم R0=c/H0 . آخه فقط توی جهان تک جزئی انحنا دار میتونستیم بگیم t0=1/H0 . یعنی آر صفر توی معادله ی فریدمان یه تعریفه ؟ که برای هر جهانی بذاریمش c/H0 ؟
یه سوال دیگه :
اگه ناظر ما سیاره اش روی لبه ی عالم باشه فقط نصف آسمون رو داره ؟ :دی اون ورش که دیگه چیزی نیست :دی مثلا همون مورچه هه که تو مادرن گفته اگه سرشو بگیره بالا چی می بینه ؟
این سوال از اینجا پیش اومد که با خودم فکر کردم اگه هیچ نقطه ای تو جهان فرق نداشته باشه پس باید برا ناظر لبه ی عالم همون جوری افق تعریف شه که واسه ناظر یه جایی جز لبه تعریف میشه ولی هیچ ایده ای راجع به جهان قابل مشاهده برای ناظر لبه ی عالم ندارم

fatemeh.zar
10-26-2014, 02:59 PM
سلام
لبه ی عالمی وجود نداره که درباره ی اینکه ناظری که اونجاس چی می بینه بحث کنیم! فکر کنم توی تاپیک انفجار بزرگ حسابی در این باره بحث شده.
http://forum.avastarco.com/forum/showthread.php?1356-%D8%A7%D9%86%D9%81%D8%AC%D8%A7%D8%B1-%D8%A8%D8%B2%D8%B1%DA%AF/
اون مورچه هم نمی تونه سرش رو بالا بگیره! چون داره توی یک دنیای 2 بعدی زندگی می کنه و کل جهانش سطح کره هست. همونطوری که ما محدودیم به اینکه فقط 3 بعد رو ببینیم.
سوال های دیگه ات رو هم می سپارم به اونایی که کیهان شناسی شون خوبه. من تازه تو دوره فهمیدم برداشتم از فاکتور مقیاس اشتباه بوده!:angry:

tina
10-27-2014, 05:49 PM
سلام !
مفهوم پارامتر چگالی منفی چیه ؟ :| مگه چگالی میتونه منفی باشه که پارامتر چگالی منفی باشه ؟ :|
(صفحه 115 رایدن (http://up.avastarco.com/images/kqw9v4ljdwd8i2k3mkl3.jpg))

ThE sTaR
10-29-2014, 12:44 AM
سلام!
خوب ما نسبت به انرژی تاریک هیچ شهودی نداریم اصلا نمیدونیم چی هست!
صرفا توی کیهان شناسی میان یه معادله مینویسن به صورت ریاضی بررسیش میکنن تو ریاضی هم یه متغیر میتونه هم مثبت هم منفی و هم صقر باشه.
و این رو هم در نظر بگیر که به چگالی یه پارامتر با یه تعریف خاصیه.دلیل نمیشه چون واسه ماده و تابش که ما میشناسیم مثبته واسه ماده تاریک که ما نمیشناسیم هم مثبت باشه.

m.Sadat
10-29-2014, 04:52 PM
درواقع انرژی تاریک یه جمله است که صرفا توی نسبیت عام تعبیرش به هندسه فضا رمان برمیگرده نه به دینامیک اون که اون جمله رو میشه ادردش توی بخش دینامیکی معادله انیشتین و ازش به عنوان یه سیال یاد کرد پس این سیالی که خودمون ساختیم ممکنه چگالی یا فشار منفی هم داشته باشه در واقع اگه چگالی منفی بشه اون اثر انرژی تاریک که کمک به انبساط شتابداره از بین میره و اثرش به انبساط کند شونده تبدیل میشه

tina
11-07-2014, 02:17 PM
سلام
من ته راه حلم (http://up.avastarco.com/images/1t98nza5vo562m3dbywr.jpg) برای سوال 5.2 رایدن (http://up.avastarco.com/images/f7099qkisww7et5g6oc.jpg)به صفر میرسم .:banghead:
اشکال کارم کجاس ؟
تازه جواب چرای آقای انصارین رو هم بلد نیستم

ali zeynali
11-07-2014, 06:51 PM
سلام
من ته راه حلم (http://up.avastarco.com/images/1t98nza5vo562m3dbywr.jpg) برای سوال 5.2 رایدن (http://up.avastarco.com/images/f7099qkisww7et5g6oc.jpg)به صفر میرسم .:banghead:
اشکال کارم کجاس ؟
تازه جواب چرای آقای انصارین رو هم بلد نیستم

شما اونجا که زمان تابش رو بر حسب t(0) می‌نویسی بعدش ازش مشتق میگیری, انتقال به سرخ رو متغییر نمیگیرید. در واقع اون فرمول همان فرمول اولیه ای است که ازش مشتق گرفتید ولی چون اینجا z رو ثابت گرفتید, میتونستید همونجا بفهمید که جوابتون آخرش صفر میشه.

Arefe
11-07-2014, 07:54 PM
توي جوابت اون قسمتي كه نوشتي dt/dt0 اشتباهه
(dt/dt0 =1/(1+z
با اين در مياد
اثباتش هم تو مادرن هست هم رايدن
براي عالم هم شعاعي تعريف نميكنيم فقط مقياس هايي از شعاع براي عالم وجود داره.و هيچ مركزي هم براي جهان وجود نداره
توي هر نقطه اي ميتوني خودت رو مركز بدوني و براش مقياسي از شعاع به دست بياري

tina
11-07-2014, 08:50 PM
الان زد رو هم توی رابطه واسه زمان تابش تغییر دادم و به یک نتیجه ی خیلی شگرف (http://up.avastarco.com/images/vqfus4mio1zorbcf9m2.jpg)در عرصه ی کیهان شناسی نائل اومدم :|
( الان راهنمایی نکنین ؛ عادت کردم هی حل نشه بیام اینجا بپرسم ، یا اینو خودم حل میکنم یا حل نمیکنم بعدا میاد 0 میشم . )

fazixa
12-10-2014, 07:11 PM
سلام کسی میتونه به این سوال جواب بده؟؟

http://s5.picofile.com/file/8156434450/xxxx.png

black.body
12-15-2014, 11:53 AM
لطفا راهنمایی کنید :
سیاره فرضی به شعاع 10 هزار کیلومتر در فاصله ی 0.5 واحد نجومی از خورشید قرار دارد و هر 32 ساعت به دور خود می گردد ، به طوریکه جهت چرخش آن برعکس جهت چرخش آن به دور خورشید است . صفحه ی استوای آن با صفحه ی چرخشش به دور خورشید هم صفحه است . شب در استوای این سیاره چقدر از روز آن بلند تر است ؟
راه آسون تر از بردار داره ؟

ali zeynali
12-27-2014, 07:32 PM
لطفا راهنمایی کنید :
سیاره فرضی به شعاع 10 هزار کیلومتر در فاصله ی 0.5 واحد نجومی از خورشید قرار دارد و هر 32 ساعت به دور خود می گردد ، به طوریکه جهت چرخش آن برعکس جهت چرخش آن به دور خورشید است . صفحه ی استوای آن با صفحه ی چرخشش به دور خورشید هم صفحه است . شب در استوای این سیاره چقدر از روز آن بلند تر است ؟
راه آسون تر از بردار داره ؟
فرض کنید نقطه ای خورشید دقیقا بالای سرش است. حساب کنید برای اینکه خورشید در حال طلوع یا غروب باشد ناظر با این نقطه چند درجه فاصله دارد؟(از دید مرکز سیاره) بعد ببینید میشه این زاویه و دوره تناوب هلالی خورشید رو
به خواسته مساله ربط بدید!؟ :)

black.body
12-27-2014, 11:05 PM
فرض کنید نقطه ای خورشید دقیقا بالای سرش است. حساب کنید برای اینکه خورشید در حال طلوع یا غروب باشد ناظر با این نقطه چند درجه فاصله دارد؟(از دید مرکز سیاره) بعد ببینید میشه این زاویه و دوره تناوب هلالی خورشید رو
به خواسته مساله ربط بدید!؟ :)
ممنون :) می نویسم اگر نشد باز می پرسم .
یک سوال دیگه : مجموعه ی استاد صدرالدینی ، صفحه ی 3 ، سوال 4 :
رصدگری در نقشه ی آسمان یک سیستم دوتایی می بیند ...
میشه حلشو بگید ؟

alighodrati
12-31-2014, 07:30 PM
سلام.این سوال مدتی است ذهن ما درگیر کرده.لطفا راه حل آن را بگویید
<<شبانه روز خورشیدی مدت زمانی است که از دید یک ناظر خورشید دوبار عبور کند .سال مدت زمانی است که سیاره یک دور به دور خورشید بگردد.اگر فرض کنیم دوره وضعی و انتقالی سیاره زهره با هم برابر باشد شبانه روز خورشیدی زهره چند سال است؟>>

arashgmn
01-02-2015, 02:41 PM
سلام.این سوال مدتی است ذهن ما درگیر کرده.لطفا راه حل آن را بگویید
<<شبانه روز خورشیدی مدت زمانی است که از دید یک ناظر خورشید دوبار عبور کند .سال مدت زمانی است که سیاره یک دور به دور خورشید بگردد.اگر فرض کنیم دوره وضعی و انتقالی سیاره زهره با هم برابر باشد شبانه روز خورشیدی زهره چند سال است؟>>

روی سطح زهره یک شاخص در نظر بگیرید که در لحظه‌ی خاصی، رو به خورشید قرار گرفته. بعد بگذارید زمان بگذره و جایجایی شاخص رو بررسی کنید. باید تعیین کنید که جه مدت بعد، شاخص دوباره به سمت خورشید قرار می‌گیره. از سرعت زاویه‌ای استفاده کنید.

پ.ن: در صورت سوال باید گفته بشه که جهت دوران وضعی و انتقالی زهره یکسانه یا مخالف هم. چون در جواب تاثیر می‌ذاره. واقعیت این که جهت این دو تا دوران (بر خلاف بقیه سیارات منظومه) مخالف هم دیگه است. به همین خاطره که خورشید در ناهید، از غرب طلوع می‌کنه و در شرق غروب! تو استاری نایت یا استلاریوم می‌تونید چک کنید...

Benyamin.sh
01-04-2015, 11:54 AM
سلام.شاید سوالم خیلی به این انجمن ربطی نداشته باشه ولی لطفا جواب بدید.
چرا جذر و مد در دوطرف کره ی زمین همزمان اتفاق میوفته؟

محمد فتحی
01-04-2015, 06:51 PM
سلام.شاید سوالم خیلی به این انجمن ربطی نداشته باشه ولی لطفا جواب بدید.
چرا جذر و مد در دوطرف کره ی زمین همزمان اتفاق میوفته؟

خوب به خاطر این که جاذبه ماه بر هر دو طرف در یک زمان اثر داره.

Benyamin.sh
01-05-2015, 10:23 AM
خوب به خاطر این که جاذبه ماه بر هر دو طرف در یک زمان اثر داره.
میتونید یه کم بیشتر توضیح بدید؟باتشکر

ali zeynali
01-05-2015, 10:23 AM
سلام.شاید سوالم خیلی به این انجمن ربطی نداشته باشه ولی لطفا جواب بدید.
چرا جذر و مد در دوطرف کره ی زمین همزمان اتفاق میوفته؟

بدلیل شتاب نسبی آب ها نسبت به زمین است.برای درک بهتر فرض کنید شتاب بصورت خطی به فاصله زمین تا ماه ربط دارد! خب 3 نقطه رو در نظر بگیرید. یکی سطح آب نزدیک به ماه(1), مرکز زمین(2) , دروترین سطح آب از ماه(3). حالا طبق فرض اولیه نقطه 1 نسبت به نقطه دو با یک شنابی دارد دور می شود. و همچنین نقطه 2 نسبت به نقطه 3 با همان شتاب دارد دور می شود. پس اگر از دید ناظران روی زمین نگاه کنید(نقطه 2) , سطح آب که نقاط 1و3 هستند هر دو دارای شتاب دور شونده نسبت به ما هستند. :)

Benyamin.sh
01-06-2015, 11:31 AM
[QUOTE=ali zeynali;77987]بدلیل شتاب نسبی آب ها نسبت به زمین است.برای درک بهتر فرض کنید شتاب بصورت خطی به فاصله زمین تا ماه ربط دارد! خب 3 نقطه رو در نظر بگیرید. یکی سطح آب نزدیک به ماه(1), مرکز زمین(2) , دروترین سطح آب از ماه(3). حالا طبق فرض اولیه نقطه 1 نسبت به نقطه دو با یک شنابی دارد دور می شود. و همچنین نقطه 2 نسبت به نقطه 3 با همان شتاب دارد دور می شود. پس اگر از دید ناظران روی زمین نگاه کنید(نقطه 2) , سطح آب که نقاط 1و3 هستند هر دو دارای شتاب دور شونده نسبت به ما هستند. :)
خیلی ممنون

Benyamin.sh
01-14-2015, 09:21 AM
سلام.من طول روز رو که برای اول تابستون و زمستون حساب میکنم طول روز اول زمستون بیشتر از تابستون میشه!هزار بارم حساب کردم.برای میل 23.5-(اول زمستون)در میاد 14.5 ساعت و برای میل 23.5 خورشید(اول تابستون)در میاد 9 ساعت و خورده ای در حالی که باید برعکس باشه.میشه بگید اشتباهم کجاس؟

Mostafa
01-14-2015, 12:24 PM
سلام.من طول روز رو که برای اول تابستون و زمستون حساب میکنم طول روز اول زمستون بیشتر از تابستون میشه!هزار بارم حساب کردم.برای میل 23.5-(اول زمستون)در میاد 14.5 ساعت و برای میل 23.5 خورشید(اول تابستون)در میاد 9 ساعت و خورده ای در حالی که باید برعکس باشه.میشه بگید اشتباهم کجاس؟

سلام به شما

خب تا روش محاسبه تون رو نگید که نمیشه فهمید اشتباه کار کجاست ! :)

قاعدتآ شما دارید طول شب رو محاسبه می کنید نه روز

اگر از این فرمول استفاده می کنید :

t=2/15 Arc Cos ( -tg phi. tg delta )l

و میل خورشید و عرض جغرافیایی رو درست قرار بدید ، باید درست بدست بیاد

اگر هم روش دیگه ای دارید بگید تا اشکالش پیدا بشه

موفق باشید :)

tina
01-23-2015, 03:02 PM
سلام
توی سوال 5.2 (http://up.avastarco.com/images/nnnukjl3a2spsbi4jayn.jpg) (فصل 5 سوال 2 )گلگر برای اینکه به جواب برسم مجبورم جوابمو در دو ضرب کنم ولی توی سوال 5.4 (http://up.avastarco.com/images/5x77g5qfrxb4w1rrilh.jpg) نه .
چرا ؟ مگه چه فرقی دارن ؟

ali zeynali
01-24-2015, 12:52 PM
سلام
توی سوال 5.2 (http://up.avastarco.com/images/nnnukjl3a2spsbi4jayn.jpg) (فصل 5 سوال 2 )گلگر برای اینکه به جواب برسم مجبورم جوابمو در دو ضرب کنم ولی توی سوال 5.4 (http://up.avastarco.com/images/5x77g5qfrxb4w1rrilh.jpg) نه .
چرا ؟ مگه چه فرقی دارن ؟
اگه ميتونيد، راه حلتون رو بزاريد ببينيم كجا مشكل وجود داره.

tina
01-24-2015, 03:07 PM
اگه ميتونيد، راه حلتون رو بزاريد ببينيم كجا مشكل وجود داره.

اون لینکا راه حلامه :)

alighodrati
01-27-2015, 06:05 PM
انرژي لازم براي فرار از سطح زمين چند برابر انرژي لازم براي فرستادن همان جسم به بيرون از منظومه شمسي است؟
لطفا راه حل ان را كامل بگوييد

starscream4002
01-28-2015, 06:30 AM
انرژي لازم براي فرار از سطح زمين چند برابر انرژي لازم براي فرستادن همان جسم به بيرون از منظومه شمسي است؟
لطفا راه حل ان را كامل بگوييد

سلام،

راه حل کامل؟!
انرژی لازم برای فرار از سطح زمین یا فرار از گرانش هر جسمی برابر مقدار انرژی جنبشی است که باید به آن بدهیم تا در مداری سهموی از آن دور شود. که این مقدار برابر است با gm/r که r فاصله از جسم مورد نظر و m هم جرم آن جسم می باشد.
حالا برای فرار از منظومه شمسی، باید از فاصله 1 واحد نجومی از خورشید فرار کند. جرم می شود جرم خورشید و فاصله هم یک واحد نجومی. برای فرار از سطح زمین هم فاصله می شود شعاع زمین و جرم هم جرم زمین.
حالا باید نسبت این مقادیر به دست آمده را حساب کنی.

موفق باشید،

س.ا.

tina
02-01-2015, 10:17 AM
سلام
من از پارسال تا الان یه قسمتی از فیلیپس رو نمیفهمم :| یه سوال بود که توش اثبات میشد وقتی پیش ستاره توی مرحله ی رمبش آرومه فشار با چگالی به توان چهار سوم متناسبه پس طبق معادله 1-15 فیلیپس ، ویریال براش کا به علاوه ی یو هست نه دو کا به علاوه ی یو ... پس چرا صفحه ی 29 قضیه ویریال رو به صورت دو کا به علاوه ی یو نوشته ؟
یه سوال دیگه هم دارم : توی رمبش آروم ذرات نسبیتی هستن ؟

Benyamin.sh
02-01-2015, 04:59 PM
سلام به شما

خب تا روش محاسبه تون رو نگید که نمیشه فهمید اشتباه کار کجاست ! :)

قاعدتآ شما دارید طول شب رو محاسبه می کنید نه روز

اگر از این فرمول استفاده می کنید :

t=2/15 Arc Cos ( -tg phi. tg delta )l

و میل خورشید و عرض جغرافیایی رو درست قرار بدید ، باید درست بدست بیاد

اگر هم روش دیگه ای دارید بگید تا اشکالش پیدا بشه

موفق باشید :)




سلام.تا اونجایی که من میدونم طول روز کامل از دوبرابر کردن زاویه ساعتی خورشید در اون روز بدست میاد پس از این فرمول رفتم:


2cosHA=tan phi *tan delta


که در اون عرض جغرافیاییم رو 36 قرار دادم.آیا راه حلم غلطه؟خیلی ممنون از توجهتان

Mostafa
02-01-2015, 05:12 PM
سلام.تا اونجایی که من میدونم طول روز کامل از دوبرابر کردن زاویه ساعتی خورشید در اون روز بدست میاد پس از این فرمول رفتم:


2cosHA=tan phi *tan delta


که در اون عرض جغرافیاییم رو 36 قرار دادم.آیا راه حلم غلطه؟خیلی ممنون از توجهتان

خب این فرمولی که نوشتید هم شکل دیگری از همانی است که خدمتتان گفتم

منتها توجه کنید که به این صورت شما زاویه ساعتی طلوع تا غروب را به دست میارید و برای تبدیلش به زمان باید به 15 تقسیمش کنید

اگر با این روش محاسبه کنید نباید مشکلی پیش بیاد

alighodrati
02-04-2015, 08:24 PM
سلام کسی میتوته سوال 6 فصل 2 نجوم کروی اسمارت راه حلشو بگه

Mostafa
02-04-2015, 09:14 PM
سلام کسی میتوته سوال 6 فصل 2 نجوم کروی اسمارت راه حلشو بگه
سلام به شما
همه دوستان مثل شما در بحبوحه المپياد و حل مسائل نيستند و ممكن است به كتاب اسمارت دسترسي نداشته باشند
اگر زحمت بكشيد و صورت سوال را بنويسيد ، راحت تر مي توانند به شما كمك كنند :)
موفق باشيد

mohammadmehdi
02-06-2015, 12:35 PM
سلام
یه سوال هندسی داشتم
مساحت مشترک بین دو عرقچین چجوری بدست میاد؟
حجم عرقچین چطور؟

tina
02-06-2015, 08:50 PM
سلام .
مساحت مشترک بین دو عرقچین (http://s4.picofile.com/file/8168990676/%D9%85%D8%B3%D8%A7%D8%AD%D8%AA_%D9%85%D8%B4%D8%AA% D8%B1%DA%A9_%D8%AF%D9%88_%D8%B9%D8%B1%D9%82%DA%86% DB%8C%D9%86.pdf.html)
حجم کره چجوری به دست میاد ؟ انتگرال داره دیگه ! منهای یه انتگرال دیگه کنی قسمت بالاش حجم عرقچین میشه ... ( خودت حساب کنی بیشتر تو ذهنت میمونه یه درصد هم فکر نکن من تنبلم نمیخوام پاشم برم اتاق بغلی اسکن کنم جواب رو )

tina
02-07-2015, 09:08 PM
سلام برای این سوال (http://up.avastarco.com/images/x5bf8x2f67zbgn38ag8.jpg)راه من اینه (http://up.avastarco.com/images/ts1hepok0juii4e4n84w.jpg) ولی جواب گزینه ی 3 هست . چرا ؟

Arefe
02-07-2015, 11:25 PM
سلام برای این سوال (http://up.avastarco.com/images/x5bf8x2f67zbgn38ag8.jpg)راه من اینه (http://up.avastarco.com/images/ts1hepok0juii4e4n84w.jpg) ولی جواب گزینه ی 3 هست . چرا ؟

دقت كن كه يه مستطيل روي كره سماوي تشكيل ميشه
ببين تو اومدي اضلاع رو در هم ضرب كردي تا مساحت مثلث رو به دست بياري ولي بايد مستطيل رو به دو مثلث تبديل ميكردي و مساحت " مثلث هاي كروي " رو به دست مياوردي كه رابطش جمع سه ضلع منهاي پي هست

tina
02-08-2015, 09:03 AM
آهاا ... آخه توی آزمون طلاهای امسال از این تقریب استفاده کرده بود گفتم شاید اینجا هم میشه :دی مرسی ^______^

سلام کسی میتوته سوال 6 فصل 2 نجوم کروی اسمارت راه حلشو بگه
سلام ؛ فک کنم راه حلش اینه (http://up.avastarco.com/images/74s2izo2cgl6f8per6vj.jpg)

amir navid
02-09-2015, 07:39 PM
ببخشید چندتا سوال داشتم اگه میشه جواب بدید:سوالای ۸و۱۱و۱۲و۲۳ تو فایل لینکی گذاشتم هست لطفن کمک کنید:http://s4.picofile.com/file/8169659968/1step.pdf.html

شروین حکیمی
02-09-2015, 11:13 PM
سلام!
من جواب دوتا سوالش را به صورت زیر نوشتم، شرمنده کمی بد خط شده :-)

http://8pic.ir/images/td1mgyqkh8yq982a0i81.png (http://8pic.ir/)
http://8pic.ir/images/r6z6ha3htbbkrg7ewk7c.jpg (http://8pic.ir/)

Arta.kh
02-11-2015, 07:32 PM
دوستان بعد از مدت ها سلام.
امکانش هست کسی "فزونی رنگ" رو برای من توضیح بده؟
ممنون میشم :)

tina
02-12-2015, 07:26 AM
دوستان بعد از مدت ها سلام.
امکانش هست کسی "فزونی رنگ" رو برای من توضیح بده؟
ممنون میشم :)
سلام :78:
رنگ یعنی اختلاف بین قدرای یه ستاره توی دو تا بازه طول موج مثلا اختلاف بین قدر آبی و مرئی ( صفحه 22 گلگر خیلی خوب گفته )
فزونی رنگ یعنی باید برای یه ستاره ی مشخص یه رنگی رو ببینیم ، اما یه رنگ دیگه رو میبینیم . علتشم فک کنم جذب نور ستاره از ستاره تا زمین باشه

Arta.kh
02-12-2015, 09:43 PM
سلام :78:
رنگ یعنی اختلاف بین قدرای یه ستاره توی دو تا بازه طول موج مثلا اختلاف بین قدر آبی و مرئی ( صفحه 22 گلگر خیلی خوب گفته )
فزونی رنگ یعنی باید برای یه ستاره ی مشخص یه رنگی رو ببینیم ، اما یه رنگ دیگه رو میبینیم . علتشم فک کنم جذب نور ستاره از ستاره تا زمین باشه

بسیار متشکرم همین امروز فهمیدم :دی
مرسی مرسی :j

tina
02-12-2015, 11:20 PM
سلام .
توی این سوال (http://up.avastarco.com/images/kk1pl2bgh0qdb27tr2v0.jpg)خود پاسخنامه اش با راه نیرو رفته و جوابش http://up.avastarco.com/images/rhhfdv321lu16rxr5udz.jpgشده ولی من وقتی میخوام از انرژی برم (قسمت اول راه حلم (http://up.avastarco.com/images/8xx1weovzfjqcmjbwtl.jpg)، قسمت دوم راه حلم (http://up.avastarco.com/images/05cmeiv1t3arscby2hk.jpg) ) جواب رو اشتباه در میارم ولی نمیفهمم اشکال کارم کجاس ...
میشه راهنمایی کنین ؟

Ehsan
02-13-2015, 12:49 AM
تو راه حلتون، چرا w0 باید رادیکال GM/R^3 باشه وقتی نیرو کاملا گرانشی نیست؟

tina
02-13-2015, 10:23 AM
تو راه حلتون، چرا w0 باید رادیکال GM/R^3 باشه وقتی نیرو کاملا گرانشی نیست؟
اون قسمتشو اشتباه نوشتم اول فک میکردم باید فقط نیروی گرانش رو به حساب بیارم :9:

ولی بازم قسمت دومش غلطه :|

Ehsan
02-13-2015, 11:07 AM
اون قسمتشو اشتباه نوشتم اول فک میکردم باید فقط نیروی گرانش رو به حساب بیارم :9:

ولی بازم قسمت دومش غلطه :|

اونم به این خاطره که اول اولش که دارید پتانسیل موثر رو می‌نویسید، برای پتانسیل نیروی فنری باید بگذارید 1/2Ar^2 که شما گذاشتید Ar^2 به خاطر همین هم آخرش یه ضریب دو زیاد آوردید.

tina
02-13-2015, 12:41 PM
آهاا :دی
برم ببینم میتونم بالاخره به جواب برسونمش یا نه ...
مرسی ^______^

Arta.kh
02-15-2015, 03:17 PM
سلام :)
یه سوال.
از دوره ی نهم، سوال 25 :)

https://photos-4.dropbox.com/t/2/AACriZbd6LzUekrQunZTzDTAw1kIaI50EsGKi7_npfCSKg/12/195216324/png/1024x768/3/1424005200/0/2/Screenshot%202015-02-15%2015.00.19.png/CMSHi10gASACIAMoASgC/uFHIObEoNPOLSINk3u5Qdicay0EWDhk_5V0Q-HA5O94

راه حل :

https://photos-3.dropbox.com/t/2/AAC1HUSNOutjVf9j7aqwM578FBK0hv2-F8kwZUeDdbqE8w/12/195216324/png/1024x768/3/1424008800/0/2/Untitled.png/CMSHi10gASACIAMoASgC/D2QMHCRqlqy0ij4kLeJ_aEzi9ScQVu18VAOMhIPe8mI

چون تداخل سازنده رخ میده ، پس وتر ها ضریب صحیحی از طول موج باشن و میدونیم که اولین بیشینه در 3 درجه رخ میده. و در پاسخنامه نوشته باید وتر 6 درجه رو ضربدر دو کنیم و از وتر 3 درجه کم کنیم. من دو برابر کردنشو نمیفهمم. برای چی باید دو برابر کنیم؟
ممنون :دی

یکی لطفن جواب بده! :(

logo64
02-16-2015, 01:20 AM
من به المپیاد علاقه خاصی دارم چطور می تونم امتحانش رو بدم ؟ ورودی داره چطوریاست ؟

tina
02-23-2015, 08:13 PM
سلام
متد سوال 86 قسمت الف (http://up.avastarco.com/images/ww9478fa5t2vnah8cic.jpg)، یه فرمول داده که من باهاش چند تا مشکل دارم :
1- مگه توی فرمولی که مادرن داده ،اینو میگم (http://up.avastarco.com/images/y03icqd0if3y2h0uu8pd.jpg)، سرعت شعاعی نسبت به سرعت خورشید نیست ؟ ( یعنی اونی که مشاهده میکنیم ) پس چرا متد سرعتی که مشاهده میکنیم رو به علاوه سرعتای دیگه میکنه ؟
2-چرا سمت راست فرمول یه کسینوس دویِ تتا اضافه تر از مادرن داره ؟

فک کنم جوابشو فهمیدم :
1- خود صورت سوال گفته جمع اون دوتا سرعتیه که ما مشاهده میکنیم.
2- توی مادرن فرض شده عرض کهکشانی نداره .

alighodrati
02-24-2015, 09:27 PM
سلام
در نجوم كروي مثلا در يه بازه زماني معين بعد يه ستاره مشخص فرقي ميكنه يا بعد هر ستاره هميشه ثابته؟ اگه ثابته محل اعتدال بهاري در كره اي كه ناظر وسط است كجاست؟
با تشكر

Mostafa
02-25-2015, 12:22 PM
سلام
در نجوم كروي مثلا در يه بازه زماني معين بعد يه ستاره مشخص فرقي ميكنه يا بعد هر ستاره هميشه ثابته؟ اگه ثابته محل اعتدال بهاري در كره اي كه ناظر وسط است كجاست؟
با تشكر
سلام
بعد و میل ستاره ها با دقت خوبی همواره ثابته
اما اون چیزی که تغییر می کنه زاویه ساعتی اونها است که بر اثر گردش زمین این تغییر به وجود میاد
محل نقطه اعتدال بهاری رو هم با توجه به زمان نجومی محلی میشه تعیین کرد

alighodrati
02-25-2015, 08:01 PM
ببخشيد دوباره سوال ميكنم ولي مگه اعتدال بهاري محل برخورد دايره البروج با استواي سماوي نيست و از اون جايي كه اين مكان يه نقطه است و ستارگان در مدارشان در حال حركت اند پس بعد بايد مدام درحال تغيير باشد، نه؟

Arta.kh
03-04-2015, 04:34 PM
سلام!! چند تا سوال :دی

https://photos-4.dropbox.com/t/2/AABiz8yxkDe7rFoU0zzPX-LCRK2Pd3BoR2gGSfSsA1PKTA/12/195216324/png/1024x768/3/1425477600/0/2/Screenshot%202015-03-04%2016.17.49.png/CMSHi10gASACIAMoASgC/jrNvp8CqL7kNoh0Me5WAq_ySwD91zXLd1lfHsB1axRg

برای اون دمای کوانتوم هم نمیفهمم چجوری بدست اومده!

ممنون میشم جواب بدین :smile:

m.Sadat
03-08-2015, 03:47 PM
سلام!! چند تا سوال :دی

https://photos-4.dropbox.com/t/2/AABiz8yxkDe7rFoU0zzPX-LCRK2Pd3BoR2gGSfSsA1PKTA/12/195216324/png/1024x768/3/1425477600/0/2/Screenshot%202015-03-04%2016.17.49.png/CMSHi10gASACIAMoASgC/jrNvp8CqL7kNoh0Me5WAq_ySwD91zXLd1lfHsB1axRg

برای اون دمای کوانتوم هم نمیفهمم چجوری بدست اومده!

ممنون میشم جواب بدین :smile:

عکسی که گذاشتید لود نمیشه اگه میشه یه باز دیگه بذارید

tina
03-11-2015, 06:37 PM
سلام توی این شکل واسه عدسی گرانشی
http://up.avastarco.com/images/lz8zrp9frkeg0384hqg.jpg
چرا این رابطه رو نوشته ؟ http://up.avastarco.com/images/tw624o6vmqdruk2qwjoj.jpg
مگه توی سه بعد نیست ؟ من فک میکنم اگه روی یه خط باشن میتونیم اینو بنویسیم

tina
03-14-2015, 03:10 PM
سلام توی این شکل واسه عدسی گرانشی
http://up.avastarco.com/images/lz8zrp9frkeg0384hqg.jpg
چرا این رابطه رو نوشته ؟ http://up.avastarco.com/images/tw624o6vmqdruk2qwjoj.jpg
مگه توی سه بعد نیست ؟ من فک میکنم اگه روی یه خط باشن میتونیم اینو بنویسیم
خودم یه جوابی حدس زدم ... اول راه حل گفته برای آلفا های کوچیک که در این صورت میشه تقریب خطی بودن رو زد ... :/ البته مطمئن نیستم ( خودم سوال میکنم خودم جواب میدم :| )

Arta.kh
03-15-2015, 04:50 PM
عکسی که گذاشتید لود نمیشه اگه میشه یه باز دیگه بذارید

سلام!!! خیلی ممنون از توجهتون! فهمیدمش مرسی :دی

ali zeynali
03-17-2015, 03:44 PM
خودم یه جوابی حدس زدم ... اول راه حل گفته برای آلفا های کوچیک که در این صورت میشه تقریب خطی بودن رو زد ... :/ البته مطمئن نیستم ( خودم سوال میکنم خودم جواب میدم :| )

سلام
بله برای آلفا و تتا های کوچیک نوشته شده این رابطه. در واقع عمل لنزینگ برای زاویه ای خیلی کوچیک اطراف یک منبع پر جرم مشاهده میشه.

tina
03-18-2015, 04:09 PM
سلام
بله برای آلفا و تتا های کوچیک نوشته شده این رابطه. در واقع عمل لنزینگ برای زاویه ای خیلی کوچیک اطراف یک منبع پر جرم مشاهده میشه.
الان فهمیدم گویا همه اون زاویه ها بردار بودن ... فقط دیدن پررنگ تر بودنشون چشم عقاب میخواسته :|

ali zeynali
03-19-2015, 01:16 AM
الان فهمیدم گویا همه اون زاویه ها بردار بودن ... فقط دیدن پررنگ تر بودنشون چشم عقاب میخواسته :|

بردار؟؟؟؟؟؟

tina
03-19-2015, 11:32 AM
بردار؟؟؟؟؟؟

بله این جوری بوده یعنی جهت داشته ...http://up.avastarco.com/images/qgt3gom85u285ztw42n.png

ali zeynali
03-20-2015, 02:17 PM
بله این جوری بوده یعنی جهت داشته ...http://up.avastarco.com/images/qgt3gom85u285ztw42n.png
:39::39::39:

tina
03-23-2015, 04:42 PM
سلام
یه سوال داشتم اگه گاز یه ستاره تک اتمی ایده آل باشه اما ستاره کاملا همرفتی باشه گاما چهار سومه یا پنج سوم ؟
(فصل 6 استلار گفته اگه ستاره توش همرفت اتفاق بیفته گاما بین چهار سوم و پنج سومه ... یه سوال توی آزمونای فاینال دوره نه هست ( همون لین امدنه ) من نمیدونم ان رو مساوی 1.5 بگیرم یا 3 )
+خوب فک کنم همون 5/3 درسته.

ali zeynali
03-25-2015, 11:14 AM
سلام
یه سوال داشتم اگه گاز یه ستاره تک اتمی ایده آل باشه اما ستاره کاملا همرفتی باشه گاما چهار سومه یا پنج سوم ؟
(فصل 6 استلار گفته اگه ستاره توش همرفت اتفاق بیفته گاما بین چهار سوم و پنج سومه ... یه سوال توی آزمونای فاینال دوره نه هست ( همون لین امدنه ) من نمیدونم ان رو مساوی 1.5 بگیرم یا 3 )
+خوب فک کنم همون 5/3 درسته.

سلام.
برای ذرات تک اتمی غیرنسبیتی فشار با توان 5/3 چگالی تناسب دارد. این مطلب جدا از بحث همرفت است. توی ستاره ای همرفتی فشار با چگالی به توان عددی گه گاما نامیدیم تناسب دارد. بخاطر همین ذرات تک اتمی ایده آل غیرنسبیتی رو به یک گاز همرفتی با گاما 5/3 میشه تشابه داد.
اونجا هم فگر کنم باید 1.5 می گرفتیم. :)

tina
03-25-2015, 04:21 PM
آها،خیلی ممنون :)
بعد یه سوال دیگه : سوال 70 عیدانه (http://up.avastarco.com/images/vc8spmqqrb8ua25bej6i.jpg) طلاهای رومانی R=6.66 دو بار تکرار شده ولی مقدار سرعت شعاعی هاش متفاوته
باید دو تا داده رو روی نمودار کشید ؟

ali zeynali
03-25-2015, 08:29 PM
آها،خیلی ممنون :)
بعد یه سوال دیگه : سوال 70 عیدانه (http://up.avastarco.com/images/vc8spmqqrb8ua25bej6i.jpg) طلاهای رومانی R=6.66 دو بار تکرار شده ولی مقدار سرعت شعاعی هاش متفاوته
باید دو تا داده رو روی نمودار کشید ؟
یجور گفتید طلا های رومانی, حس غریبی کردم. :(
بله بنظرم منظور این بوده که از این شعاع دو بار داده گیری انجام شده است و باید دو نقطه بر روی نمودار برای این فاصله کشیده شود.

MSAM
03-29-2015, 05:32 PM
سلام نجومیای گل
میشه بگید زاویه ی آنومالی اعتدال و انقلاب چطوری پیدا میشه؟ یعنی باید برای اینکه طولانی ترین روز یا شب رو پیدا کنیم چه مشخصاتی از مدار رو بدونیم و چیکارشون کنیم؟
مرسی

ali zeynali
03-30-2015, 12:24 AM
سلام نجومیای گل
میشه بگید زاویه ی آنومالی اعتدال و انقلاب چطوری پیدا میشه؟ یعنی باید برای اینکه طولانی ترین روز یا شب رو پیدا کنیم چه مشخصاتی از مدار رو بدونیم و چیکارشون کنیم؟
مرسی

برای هر سیاره ای با هر مداری برای محاسبه آنومالی های هر تاریخی(نه لزوما انقلاب و اعتدال) لازم است شناسه حضیض مدار رو بدونیم.

MSAM
03-30-2015, 12:59 PM
مرسی
اصلا نمیدونستم شناسه ی حضیض چیه! رفتم خوندم
یه سوال دیگه:
رابطه ی دوره تناوب هلالی برای مدارهای بیضوی چطوری به دست میاد؟ میشه اثباتشو بزارید اگه دارید

tina
03-30-2015, 03:57 PM
رابطه ی دوره تناوب هلالی برای مدارهای بیضوی چطوری به دست میاد؟ میشه اثباتشو بزارید اگه دارید
میشه سوالشو بگین ؟ آقای صدرالدینی یه سوال داشتن که یه چیزی تو همین مایه ها میخواست اما نیازی به اثبات دوره تناوب هلالی نبود با یکم دقت حل میشد.(البته اگه رابطه ی کلی ای واسه دوره تناوب هلالی مدار های بیضوی وجود داره خوشحال میشم بدونم :دی)
+میشه برای این سوال (http://s5.picofile.com/file/8170947926/6.JPG) هم راهنمایی کنین ؟خودم یه راهی رفتم و مجبور شدم از رابطه هایی که F مدار هذلولی رو به تتا ربط میده استفاده کنم و تهش هم یه قسمتی رو نمیتونم رفع ابهام کنم ( فک کنم راهم غلطه چون دیگه ناجوانمردیه رفع ابهام یه چیزی در ال ان به آدم بدن :| )

MSAM
03-30-2015, 05:14 PM
میشه سوالشو بگین ؟ )
صرفا یه کنجکاوی شخصی بود!!!!! جدیدا خیلی با بیضی درگیرم... یعنی ممکنه واقعا هیچ رابطه ی خاصی نداشته باشه ؟

tina
03-31-2015, 12:45 AM
صرفا یه کنجکاوی شخصی بود!!!!! جدیدا خیلی با بیضی درگیرم... یعنی ممکنه واقعا هیچ رابطه ی خاصی نداشته باشه ؟

من که نگفتم رابطه ی خاصی نداره :دی

MSAM
03-31-2015, 12:10 PM
الان http://s4.picofile.com/file/8179953434/Untitled2.jpg رو دیدم که شاید مربوط باشه به سوال قبلیم. اینو من قبلا با همون رابطه ی دوره تناوب هلالی مدار دایروی حل کرده بودم و مطمئنم استدلال کافی برای استفاده ازش رو داشتم اما حالا گیج شدم!!!!!!!!:17::17::17:
اینو میشه با همون رابطه ی ساده حل کرد؟
اصلا بیضی چیه؟ دایره چیه؟ نجوم چیه؟ من کیم؟:serious business:

tina
04-04-2015, 06:30 PM
سلام.
اگه به ما یه تابع چگالی بده بعد بگه سرعت مدار دایره ای رو به صورت تابعی از فاصله در بیار دو تا راه داریم اول اینکه اینجوری (http://up.avastarco.com/images/onh0ayf1e445wdiubvvt.jpg) بنویسیم و جرم رو به صورت انتگرال چها پی آر دو رو دی آر حساب کنیم دومین راه هم اینه که انرژی بنویسیم که در اون صورت پتانسیل رو این جوری (http://up.avastarco.com/images/afs9n9tzdgurrc1ko4zw.jpg) مینویسیم .
ولی جوابا یکی در نمیاد :( توی راه دوم قسمتای خارجی هم پتانسیل دارن ولی توی راه اول فقط نیرو از جرم داخل وارد میشه . اشکال کار کجاس ؟
(خیلی با این قضیه که نیرو وارد نمیکنن اما پتانسیل جسم رو تغییر میدن مشکل دارن.)

ali zeynali
04-05-2015, 11:43 PM
سلام.
اگه به ما یه تابع چگالی بده بعد بگه سرعت مدار دایره ای رو به صورت تابعی از فاصله در بیار دو تا راه داریم اول اینکه اینجوری (http://up.avastarco.com/images/onh0ayf1e445wdiubvvt.jpg) بنویسیم و جرم رو به صورت انتگرال چها پی آر دو رو دی آر حساب کنیم دومین راه هم اینه که انرژی بنویسیم که در اون صورت پتانسیل رو این جوری (http://up.avastarco.com/images/afs9n9tzdgurrc1ko4zw.jpg) مینویسیم .
ولی جوابا یکی در نمیاد :( توی راه دوم قسمتای خارجی هم پتانسیل دارن ولی توی راه اول فقط نیرو از جرم داخل وارد میشه . اشکال کار کجاس ؟
(خیلی با این قضیه که نیرو وارد نمیکنن اما پتانسیل جسم رو تغییر میدن مشکل دارن.)
خب الان مشکلی هست؟ توی راه دوم برای بدست آوردن سرعت همچنان می خواین از پتانسیل مشق بگیرید دگه؟ نه؟ حالا مشتق می گیرید میگید جمله اول همون قبلی شد ولی جمله دوم نشد؟ از M(r) مشتق گرفتید؟ :)

tina
04-06-2015, 06:27 AM
خب الان مشکلی هست؟ توی راه دوم برای بدست آوردن سرعت همچنان می خواین از پتانسیل مشق بگیرید دگه؟ نه؟ حالا مشتق می گیرید میگید جمله اول همون قبلی شد ولی جمله دوم نشد؟ از M(r) مشتق گرفتید؟ :)
نه ؛نمیخوام تبدیلش کنم باز به نیرو . میخوام بنویسم نصف سرعت مساوی است با منفی نصف پتانسیل.
فک کنم فهمیدم اشکال کارم کجاس من خیلی ساده فرض کردم انرژی مدار دایره ای رو در این حالت میدونم درحالی که نمیدونم.توی گلگر هم واسه ی سهمی این کارو کرده ولی واسه ی مدار دایره ای این کارو نکرده .
مرسی :دی

tina
04-13-2015, 07:51 AM
سلام
برای این سوال (http://up.avastarco.com/images/bd1cin98lwityckjv2rb.jpg) ، راه من اینه (http://up.avastarco.com/images/ft1jbf6ti99iflnl947w.jpg) ، ولی اردر جوابم خیلی با اردری که مسئله خواسته فرق داره .
چرا ؟
+راهم غلطه چون 1- همون اول یه ترم نیرو منفی نیست و مثبته 2- نمیشه از پتانسیل نوشت چون مبدا یکی میشه توی بینهایت یکی میشه توی 0

mohammadmehdi
04-28-2015, 07:11 AM
سلام
اگه دو تا دایره عظیمه با هم یه زاویه ای داشته باشن دایره صغیره موازی با یکی از اونا هم دایره عظیمه رو با همون زاویه قطع میکنه؟!
اون زاویه رو چجوری باید به دست آورد؟

mohammadmehdi
04-28-2015, 07:26 PM
6530
تو این سوال تک تک ستاره ها یعنی هر صد هزار تا ستاره؟
اینجوری که محاله!! :-/
منظور این سوال چیه؟

va.ahmadi
04-28-2015, 11:16 PM
سلام
اگه دو تا دایره عظیمه با هم یه زاویه ای داشته باشن دایره صغیره موازی با یکی از اونا هم دایره عظیمه رو با همون زاویه قطع میکنه؟!
اون زاویه رو چجوری باید به دست آورد؟

سلام !
سوال 2 فصل 2 اسمارت رو یه نیگا بنداز ..

tina
04-29-2015, 07:59 AM
6530
تو این سوال تک تک ستاره ها یعنی هر صد هزار تا ستاره؟
اینجوری که محاله!! :-/
منظور این سوال چیه؟
این راهی که به ذهنم میرسه رو با کسی چک نکردم برای همین ممکنه غلط باشه
توی صورت سوال چیزی در مورد توزیع ستاره های خوشه نگفته فقط گفته توزیع شدن، برای همین فرض کنین جوری توزیع شدن که وقتی همه ی ستاره های خوشه رو روی صفحه ی دید تصویر میکنین به طور یکنواخت توزیع میشن (یعنی توزیعشون توی حجم خوشه جوریه که چگالی سطحی ثابته) . کره ی تصویر شده روی صفحه ی دید یه دایره با شعاع یک پارسکه تعداد ستاره هارو دارین پس میتونین فاصله ی متوسط ستاره های تصویر شده رو به دست بیارین بعدشم یک ممیز بیست و دو لاندا دی ام رو مساوی این فاصله تقسیم بر فاصله ی خوشه از ما بذارین .

va.ahmadi
04-29-2015, 10:37 AM
این راهی که به ذهنم میرسه رو با کسی چک نکردم برای همین ممکنه غلط باشه
توی صورت سوال چیزی در مورد توزیع ستاره های خوشه نگفته فقط گفته توزیع شدن، برای همین فرض کنین جوری توزیع شدن که وقتی همه ی ستاره های خوشه رو روی صفحه ی دید تصویر میکنین به طور یکنواخت توزیع میشن (یعنی توزیعشون توی حجم خوشه جوریه که چگالی سطحی ثابته) . کره ی تصویر شده روی صفحه ی دید یه دایره با شعاع یک پارسکه تعداد ستاره هارو دارین پس میتونین فاصله ی متوسط ستاره های تصویر شده رو به دست بیارین بعدشم یک ممیز بیست و دو لاندا دی ام رو مساوی این فاصله تقسیم بر فاصله ی خوشه از ما بذارین .


سلام !
کارتون تقریب خوبیه ولی فکر کنم بهتره چگالی حجمی رو ثابت بگیریم ! بعد از روی اون چگالی سطحی رو دربیاریم ( سوال 11 مرحله 1 امسال ) .. حالا فاصله اولین ستاره از مرکز ( فاصله تصویر شده روی صفحه دید ما ) رو بدست میاریم و حداکثر فاصله مون رو با استفاده از حد رایله محاسبه میکنیم .
خود سوال گفته " در شعاع موثر 1 پارسک توزیع شدن " منظورش هم میتونه چگالی حجمیه ثابت باشه هم چگالی سطحی ثابت !!

mest98
06-28-2015, 03:36 AM
من این سوال رو هرکار میکنم نمیتونم با فرض های درست به جواب درست برسونم. میشه بگید چطوری حل میشه لطفا؟
"کهکشان خودی را به صورت قرصی با ضخامت ثابت درنظر بگیرید. نشان دهید عمق نوری تاریک شدن بوسیله ی گردوغبار بین ستاره ای باید متناسب با csc b باشد که bعرض کهکشانی است."

tina
06-28-2015, 03:23 PM
من این سوال رو هرکار میکنم نمیتونم با فرض های درست به جواب درست برسونم. میشه بگید چطوری حل میشه لطفا؟
"کهکشان خودی را به صورت قرصی با ضخامت ثابت درنظر بگیرید. نشان دهید عمق نوری تاریک شدن بوسیله ی گردوغبار بین ستاره ای باید متناسب با csc b باشد که bعرض کهکشانی است."

سلام
این چیزی بود که به ذهن من رسید
http://up.avastarco.com/images/m8kpn062pwa5nzq7tp2l.jpg

mest98
06-29-2015, 05:19 AM
آره منم همینکارو کردم. نمیفهمیدم چرا از d=h.cscb نتیجه میگیریم d متناسبه با csc b!!!
حس میکردم این معادل با ثابت درنظر گرفتن h میشه و این یه فرضه غلطه! نمیدونم چرا درست نتونسته بودم بهش فکر کنم.
مرسی

moonlight13
10-17-2015, 10:52 PM
سلام
این سوال چجوری حل میشه؟
http://uupload.ir/files/pyo_14451091817951315425345.jpg

Ahmad
06-10-2016, 03:35 PM
سلام
کتاب Handbook of CCD Astronomy ترجمه اش هست یا نه؟
اگه ترجمه شده. لطفا انتشارات و جزئیاتش جهت خرید را بفرمایید